Download as pdf or txt
Download as pdf or txt
You are on page 1of 115

Everyone makes

University
University
it Nueva
of of
in Caceres
Nueva Caceres
College of
College ofEngineering
Engineeringandand
Architecture
Architecture

MATHEMATICS

Joanne Michelle B. Aguilar

We champion
excellence
______________________________________
We nurture Dreams Name of Student

We do the right
things right
______________________________________
We are dynamic Instructor
and creative

We respect each other


and work as a team

cea@unc.edu.ph
054-4726100 loc. 121
COPYRIGHT © 2020
UNIVERSITY OF NUEVA CACERES COLLEGE OF ENGINEERING AND ARCHITECTURE

CONTENTS
CONTENTS ............................................................................................................................................................. 2
COURSE DETAILS .................................................................................................................................................... 4
COURSE OUTLINE ................................................................................................................................................... 5
WEEK 1: FUNCTIONS, LIMITS, AND SLOPE OF THE TANGENT LINE TO THE CURVE .................................................. 7
I. INTRODUCTION ......................................................................................................................................................7
II. OBJECTIVES ...........................................................................................................................................................7
III. LECTURE-DISCUSSION .............................................................................................................................................7
IV. LEARNING ACTIVITY 1.1 ........................................................................................................................................15
V. EXERCISES 1.1 .....................................................................................................................................................16
WEEK 2: THE DERIVATIVE AND BASIC RULES ........................................................................................................ 17
I. INTRODUCTION ....................................................................................................................................................17
II. OBJECTIVES .........................................................................................................................................................17
III. LECTURE-DISCUSSION ...........................................................................................................................................17
IV. LEARNING ACTIVITY 2.1 ........................................................................................................................................22
V. EXERCISES 2.1 .....................................................................................................................................................23
WEEK 3: THE DERIVATIVE AND CHAIN RULE......................................................................................................... 24
I. INTRODUCTION ....................................................................................................................................................24
II. OBJECTIVES .........................................................................................................................................................24
III. LECTURE-DISCUSSION ...........................................................................................................................................24
IV. LEARNING ACTIVITY 3.1 ........................................................................................................................................27
V. EXERCISES 3.1 .....................................................................................................................................................28
WEEK 4: HIGHER ORDER DERIVATIVES AND IMPLICIT DIFFERENTIATION ............................................................. 29
I. INTRODUCTION ....................................................................................................................................................29
II. OBJECTIVES .........................................................................................................................................................29
III. LECTURE-DISCUSSION ...........................................................................................................................................29
IV. LEARNING ACTIVITY 4.1 ........................................................................................................................................35
V. EXERCISES 4.1 .....................................................................................................................................................36
WEEK 5: DERIVATIVE OF TRIGONOMETRIC FUNCTIONS ....................................................................................... 37
I. INTRODUCTION ....................................................................................................................................................37
II. OBJECTIVES .........................................................................................................................................................37
III. LECTURE-DISCUSSION ...........................................................................................................................................37
IV. LEARNING ACTIVITY 5.1 ........................................................................................................................................40
V. EXERCISES 5.1 .....................................................................................................................................................41
WEEK 6: DERIVATIVE OF INVERSE TRIGONOMETRIC FUNCTIONS ......................................................................... 42
I. INTRODUCTION ....................................................................................................................................................42
II. OBJECTIVES .........................................................................................................................................................42
III. LECTURE-DISCUSSION ...........................................................................................................................................42
IV. LEARNING ACTIVITY 6.1 ........................................................................................................................................45
V. EXERCISES 6.1 .....................................................................................................................................................47

CALCULUS 1 v.1.0 by JOANNE MICHELLE B. AGUILAR, REE 2


UNIVERSITY OF NUEVA CACERES COLLEGE OF ENGINEERING AND ARCHITECTURE

WEEK 7: DERIVATIVE OF LOGARITHMIC FUNCTIONS............................................................................................ 48


I. INTRODUCTION ....................................................................................................................................................48
II. OBJECTIVES .........................................................................................................................................................48
III. LECTURE-DISCUSSION ...........................................................................................................................................48
IV. LEARNING ACTIVITY 7.1 [20] .................................................................................................................................52
V. EXERCISES 7.1 .....................................................................................................................................................53
WEEK 8: DERIVATIVE OF EXPONENTIAL FUNCTIONS ............................................................................................ 53
I. INTRODUCTION ....................................................................................................................................................54
II. OBJECTIVES .........................................................................................................................................................54
III. LECTURE-DISCUSSION ...........................................................................................................................................54
IV. LEARNING ACTIVITY 8.1 ........................................................................................................................................56
V. EXERCISES 8.1 .....................................................................................................................................................57
WEEK 9: MIDTERM EXAM .................................................................................. ERROR! BOOKMARK NOT DEFINED.
WEEK 10: DERIVATIVE OF HYPERBOLIC AND INVERSE HYPERBOLIC FUNCTIONS .................................................. 58
I. INTRODUCTION ....................................................................................................................................................58
II. OBJECTIVES .........................................................................................................................................................58
III. LECTURE-DISCUSSION ...........................................................................................................................................58
IV. LEARNING ACTIVITY 9.1 ........................................................................................................................................62
V. EXERCISES 9.1 .....................................................................................................................................................64
WEEK 11: POLYNOMIAL CURVES .......................................................................................................................... 65
I. INTRODUCTION ....................................................................................................................................................65
II. OBJECTIVES .........................................................................................................................................................65
III. LECTURE-DISCUSSION ...........................................................................................................................................66
IV. LEARNING ACTIVITY 10.1 ......................................................................................................................................69
V. EXERCISES 10.1 ................................................................................................................................................... 71
WORKS CITED ...................................................................................................................................................... 91
APPENDIX ............................................................................................................................................................ 93
I. THE SOLUTION TO LEARNING ACTIVITIES ...................................................................................................................93

CALCULUS 1 v.1.0 by JOANNE MICHELLE B. AGUILAR, REE 3


UNIVERSITY OF NUEVA CACERES COLLEGE OF ENGINEERING AND ARCHITECTURE

Course Details

Course Code : Math112g


Course Title : Calculus 1
Units : 3 units Lec
Commitment : 3 hrs per week
Course Description : The course provides the students with basic knowledge about theorems and
principles which are effective and powerful tools in the applied fields. It is
likewise the purpose to present calculus as an important branch of
mathematical analysis. As a background, however, students must have a
working familiarity with basic mathematical concepts acquired in algebra and
trigonometry. Throughout the course, the treatment has been to make
mathematical ideas clear, concise, and competent, consistent only with the
demands for rigor, simplicity and clarity.

Prerequisites : Math111 – Mathematics for Engineers


Textbook :
References : Ref 1: Stewart, James (8th edition), 2017 CENGAGE Learning Asia Pte Ltd.
Calculus Early Transcendental Metric Edition
Ref 2: Larson, Ron and Edwards, Bruce (9th edition), 2017 CENGAGE Learning
Asia Pte Ltd. Calculus.

Couse Outcomes : After completion of the course the student should be able to:
1. Evaluate the slope of the tangent line to the curve using limit
definition.
2. Use Elementary formulas in differentiating elementary functions i.e.
algebraic, trigonometric, inverse trigonometry, exponential,
logarithmic, hyperbolic and inverse hyperbolic functions.
3. Apply the principle of differentiation in solving mathematical word
problems.

Resources Required : Calculator, Pencil, Notebook


Software Required : NA
Quizzes : Submit test booklets to the testing center before taking the examination.
Examination : Midterm and Final Examination will be administered at a testing center
nearest to your location identified by the University

Midterm Finals
Assessments Percentage Assessments Percentage
Quizzes 30% Quizzes 30%
Exercises 20% Exercises 20%
Examination 50% Examination 50%
Total 100% Total 100%
General Average 40% Midterm Ave + 60% Final Ave.
Required Gen Ave. to Pass 75%

CALCULUS 1 v.1.0 by JOANNE MICHELLE B. AGUILAR, REE 4


UNIVERSITY OF NUEVA CACERES COLLEGE OF ENGINEERING AND ARCHITECTURE

Course Outline
Study Learning Outcomes
Topic OBE Learning Activities Assessments Remarks
Period
Apply the slope of
the tangent line to
the curve formula in
The Slope of the Tangent
Week 1 finding the Practice Problems Exercises
Line to the Curve
derivatives of the
functions.

Perform
differentiation
Week 2 Derivative and Basic Rules operations using Practice Problems Exercises
the basic rules on
algebraic functions.

Perform
differentiation
Week 3 Derivative and Chain Rule operations using Practice Problems Exercises
Chain Rule on
algebraic functions.

Perform
differentiation
Higher-Order Derivatives operations on
Week 4 Practice Problems Exercises
and Implicit Functions implicit algebraic
functions.

Week 5 Prelim Exam

Perform
differentiation
The Derivative of operations on
Week 6 Practice Problems Exercises
Trigonometric Function trigonometric
functions.

Compute the
derivative of inverse
trigonometric
The Derivative of Inverse functions using
Week 7 Practice Problems Exercises
Trigonometric Function basic rules, chain
rule, and implicit
differentiation.

Compute the
derivative of a
logarithmic function
The Derivative of using basic rules,
Week 8 Practice Problems Exercises
Logarithmic Functions chain rule, and
implicit
differentiation.

Week 9 Midterm Exam

CALCULUS 1 v.1.0 by JOANNE MICHELLE B. AGUILAR, REE 5


UNIVERSITY OF NUEVA CACERES COLLEGE OF ENGINEERING AND ARCHITECTURE

Compute the
derivative of
exponential
The Derivative of functions using
Week 10 Practice Problems Exercises
Exponential Functions basic rules, chain
rule, and implicit
differentiation.

The Derivative of
Week 11 Hyperbolic and Inverse Practice Problems Exercises
Hyperbolic Functions
Week 12 Polynomial Curves Practice Problems Exercises

Week 13 Semifinal Exam


Increasing and Decreasing
Week 14 Exercises
Functions
Week 15 Related Rates Practice Problems Exercises

Week 16 Maxima and Minima Practice Problems Exercises

Week 17 Summative Test Summative Test

Week 18 Final Exam

CALCULUS 1 v.1.0 by JOANNE MICHELLE B. AGUILAR, REE 6


UNIVERSITY OF NUEVA CACERES COLLEGE OF ENGINEERING AND ARCHITECTURE

Week 1: Functions, Limits, and Slope of the Tangent Line to the


Curve

I. Introduction

To understand the slope of the tangent line to the curve, we must first recall functions, limits,
and secant lines. A secant line will intersect a curve at more than one point, where a tangent
line only intersects a curve at one point and show the direction of the curve. The first part of the
discussion will focus on the derivation of the formula for the slope of the tangent line using
functions and limits. The slope of a tangent line to a curve is the definition we use for a function
called the derivative.

II. Objectives

After completion of this course, you should be able to:

1. Differentiate functions from relations.


2. Derive the slope of the tangent line to the curve formula using limits definition.
3. Apply the slope of the tangent line to the curve formula in finding the derivatives of the
functions.

III. Lecture-Discussion

[1]“Relations and Functions" is one of the utmost significant topics in algebra. Relations and functions –
these are the two terms having dissimilar meanings mathematically. You might get tangled about their
dissimilarity.
An ordered pair represents (INPUT, OUTPUT): The relation shows the relationship between INPUT and
OUTPUT. Whereas a function is a relation that derives one OUTPUT for each given INPUT.
Note: All functions are relations, but not all relations are functions.

[1]What is a Function?

A function is a relation that defines that there should be only one output for each input. It can be a
distinct kind of relation which follows a rule i.e. every x-value should be related with only one y-value.
Let us also look at the definition of Domain and Range of a function.

Domain It’s a group of the first values in the ordered pairs (Set of all input values)
Range It is a group of the second values in the ordered pairs (Set of all output values)

Table 1.1 Domain and Range [1]

CALCULUS 1 v.1.0 by JOANNE MICHELLE B. AGUILAR, REE 7


UNIVERSITY OF NUEVA CACERES COLLEGE OF ENGINEERING AND ARCHITECTURE

Example 1.1
In the relation, {(-1, -7), {3, 5), (8, 9)},
The domain is {-1, 3, 8} and Range is {-7, 5, 9}.

Note: Don’t consider duplicates while writing Domain and Range and also write it in increasing order.

The vertical line test is a method that is used to find whether a given relation is a function or not. The
approach is simple. Draw a vertical line cutting through the graph of the relation, and then observe the
points of intersection. If the vertical line you drew intersects the graph more than once for any value of
x then the graph is not the graph of a function.

Figure 1.1 [2]

[3]Using basic algebraic functions, what limitations are there when working with real numbers?

A) You CANNOT divide by zero.


• Any values that would result in a zero denominator are NOT allowed, therefore the
domain of the function (possible x values) would be limited.
B) You CANNOT take the square root (or any even root) of a negative number.
• Any values that would result in negatives under an even radical (such as square roots)
result in a domain restriction.

Example 1.2
Find the domain of the function
3𝑘 + 10𝑘 − 8
ℎ(𝑘) =
𝑘 − 3𝑘 − 28

CALCULUS 1 v.1.0 by JOANNE MICHELLE B. AGUILAR, REE 8


UNIVERSITY OF NUEVA CACERES COLLEGE OF ENGINEERING AND ARCHITECTURE

Solution:
You can substitute any real number in the numerator, but you must avoid inputs that make the
denominator 0.

Apply the factoring method, set the 𝑘 − 3𝑘 − 28 = 0


(𝑘 + 4)(𝑘 − 7) = 0
factors to zero and solve for x.
𝑘 + 4 = 0; 𝑘 − 7 = 0
𝑘 = −4, 𝑘 = 7

The Domain of h(k)


The domain consists of the set of all real numbers except k = 4 and k = 7 or
{𝑘|𝑘  4 𝑎𝑛𝑑 𝑘  7}.

[4]Limit of a Function

The function

Figure 1.2 Graph of function

is not defined at x = 2, so its graph has a “hole” at x = 2.

Values 𝑓(𝑥) = may be computed near x = 2

Table 1.3 Table of Values

CALCULUS 1 v.1.0 by JOANNE MICHELLE B. AGUILAR, REE 9


UNIVERSITY OF NUEVA CACERES COLLEGE OF ENGINEERING AND ARCHITECTURE

The values of f(x) get closer and closer to 4 as x gets closer and closer to 2.
We say that “the limit of 𝑓(𝑥) = as x approaches 2 equals 4” and write lim .

[4]Definition of Limit of a Function

Let f be a function and let a and L be real numbers. L is the limit of f(x) as x approaches a, written
lim 𝑓(𝑥) = 𝐿

if the following conditions are met:
1. As x assumes values closer and closer but not equal to on both sides of the corresponding values
of f(x) and the latter get closer and closer but not equal to L.
2. The value of f(x) can be made as close to L as desired by taking values of x arbitrarily close to a.

[5]The Derivative and the Slope of a Graph

Straight line has a constant slope because of its linear characteristic while a curve is consists of group
points that change direction.

Hence the slope of a curve changes


from one point to another.

Therefore we speak of the slope of the


curve at a point.

Figure 1.3 Tangent Lines to the Curve [5]

The slope of the tangent line to the curve at a point is called the Derivative of the curve at that point.

CALCULUS 1 v.1.0 by JOANNE MICHELLE B. AGUILAR, REE 10


UNIVERSITY OF NUEVA CACERES COLLEGE OF ENGINEERING AND ARCHITECTURE

Remember that the derivative at the


indicated point is the slope of the tangent
line at that point.

You usually need two points on the tangent


line to calculate its slope. But you only have
one point as indicated in figure 1.4.

How can you compute the slope of the tangent


line using only one point?
Figure 1.4 Tangent line at a point [5]

Using Limits to calculate the derivative

To be able to find the slope of the tangent


line, an approximate value could be attained
from the slope of a secant line that passes
through the given point. Recall that secant
line passes at two or more points on the curve.

Figure 1.5 Secant Line [5]

The secant line cuts the curve at points (a, f(a)) and (𝑎 + ℎ, 𝑓(𝑎 + ℎ)). Note that the difference between
the x-values is h.

𝑟𝑖𝑠𝑒 𝑓(𝑎 + ℎ) − 𝑓(𝑎) 𝑓(𝑎 + ℎ) − 𝑓(𝑎)


𝑚= = =
𝑟𝑢𝑛 𝑎+ℎ−𝑎 ℎ

CALCULUS 1 v.1.0 by JOANNE MICHELLE B. AGUILAR, REE 11


UNIVERSITY OF NUEVA CACERES COLLEGE OF ENGINEERING AND ARCHITECTURE

A better approximation to the slope of the tangent


line will be attained if you choose a point closer to
the given point. Specifically if you make h smaller.
Figure 1.6 shows that the slope of the cyan secant line
is closer to the slope of the tangent line than the
original green secant line that we use. The value of h is
smaller for the cyan secant line than for the original
green secant line.

Figure 1.6 Slope of Tangent Line to the Curve at a Point [5]

As the second point gets closer to the given point (that is, as h approaches zero), then the slope of the
secant line approaches the slope of the tangent line. This gives rise to the following limit formula for the
slope of the tangent line at the point (a, f(a)):
𝑓(𝑎 + ℎ) − 𝑓(𝑎)
𝑚(𝑡𝑎𝑛) = lim = lim 𝑚(sec)
→ ℎ →

This is called the Derivative of 𝑓(𝑥) at the point where𝑥 = 𝑎, or𝑓’(𝑎). We say “f - prime of a.”

Note: The derivative of a function at a point is the slope of the tangent line to the curve at the given
point.

[6]The Derivative of a Function

To compute the derivative function, simply replace a with x in the limit definition of the derivative, then
solve for a function of x. That is, calculate:

𝑓(𝑎 + ℎ) − 𝑓(𝑎)
𝑓′(𝑥) = lim
→ ℎ

CALCULUS 1 v.1.0 by JOANNE MICHELLE B. AGUILAR, REE 12


UNIVERSITY OF NUEVA CACERES COLLEGE OF ENGINEERING AND ARCHITECTURE

Example 1.3

Determine the derivative of f ( z )  z 2

Solution

f  z  h   f z  When limit is directly applied, the


f ' ( z )  lim
h0 h solution becomes undefined.

f ' ( z )  lim
  
zh 2  f z 2  Thus we need to simplify expression
to eliminate h in the denominator. To
h0 h remove h, rationalize the numerator.
 z  h  z  z  h  z 
f ' ( z )  lim   
 z  h  z 
In multiplying, apply product of
h0
 h   sum and difference rule

f ' ( z )  lim
 zh    z
2 2
Evaluate the difference of two
h0 h zh  z squares
zhz
f ' ( z )  lim
h0 h 
zh  z  Combine similar terms

h
f ' ( z )  lim
h0 h 
zh  z  Divide common factor

1
f ' ( z )  lim
h0 zh z
1
f ' (z)  Substitute the limit
z0  z
1
f ' (z) 
z z
1
f ' (z)  Answer
2 z

Example 1.4

1 3
Find the derivative of f ( r )  r
2 5
Solution

CALCULUS 1 v.1.0 by JOANNE MICHELLE B. AGUILAR, REE 13


UNIVERSITY OF NUEVA CACERES COLLEGE OF ENGINEERING AND ARCHITECTURE

f ( r  h)  f ( r ) Apply limit derivative formula


f ' (r )  lim h0
h
1
r  h  3   1 r  3  Distribute to remove parenthesis
2 5 2 5
f ' (r )  lim h0
h
1 1 3 1 3
r h  r Combine similar terms
f ' (r )  lim h0 2 2 5 2 5
h
1
h
2 Divide common terms
f ' (r )  lim h0
h
1 Apply limit
f ' (r )  lim h0
2
1 Answer
f ' (r ) 
2

Example 1.5

Find the derivative of f ( z )  5 z  3z  7 .


2

f ( z  h)  f ( z ) Apply limit definition


f ' ( z )  lim h0
h

f ' ( z )  lim h0


  
5 z  h   3z  h   7  5 z 2  3 z  7
2
Expand binomial and distribute
h
5 z  10hz  5h  3 z  3h  7  5 z 2  3 z  7
2 2
Combine similar terms
f ' ( z )  lim h0
h
10hz  5h  3h
2
Factor out h
f ' ( z )  lim h0
h
h10 z  5h  3 Divide common factor
f ' ( z )  lim h0
h
f ' ( z )  lim h0 10 z  5h  3 Apply limit

f ' ( z )  10 z  3 Answer

CALCULUS 1 v.1.0 by JOANNE MICHELLE B. AGUILAR, REE 14


UNIVERSITY OF NUEVA CACERES COLLEGE OF ENGINEERING AND ARCHITECTURE

IV. Learning Activity 1.1


Instruction: Use the limit definition to find the derivatives of a function.

1. f ( x)  4  x  3

x 1
2. f ( x) 
2 x

2
3. f ( x)  x 3

4. f ( x)  cos 3 x

x 1
5. f ( x) 
x  3x
2

6. f ( x)  x 3  x

CALCULUS 1 v.1.0 by JOANNE MICHELLE B. AGUILAR, REE 15


UNIVERSITY OF NUEVA CACERES COLLEGE OF ENGINEERING AND ARCHITECTURE

V. Exercises 1.1

INSTRUCTION: Use the limit definition of the derivative to find the derivative of the following functions.
Use a test booklet for your solution.

1. f ( x)  6

2. f (t )  3  14t

3. f ( x)  x 2

4. f (z)  4x2  9z

5. g (t )  10  5t  t 2

6. Q( x )  2 x 3  1

7. f ( x)  x 3  2 x 2  x  1

5
8. w( x) 
x

x 1
9. f ( x) 
x4

10. f ( x )  3 x  4

CALCULUS 1 v.1.0 by JOANNE MICHELLE B. AGUILAR, REE 16


UNIVERSITY OF NUEVA CACERES COLLEGE OF ENGINEERING AND ARCHITECTURE

Week 2: The Derivative and Basic Rules

I. Introduction

[7]The derivative of a function of a real variable calculates the sensitivity to change of the function
value or output value with respect to a change in its argument or input value. Derivatives are essential
instrument of calculus.

II. Objectives

After completion of this course, you should be able to:

1. Perform differentiation operations using the basic rules on algebraic functions.


2. Apply basic algebraic concepts and operations in simplifying derivatives of algebraic
functions.

III. Lecture-Discussion

[8]Notation for Differentiation

 The derivative of 𝑓(𝑥) is denoted𝑓 ‘(𝑥).


 Another notation is called Leibniz notation, the derivative of y with respect to x is .
Thus,

 Other notations for the derivative (with respect to x) of a function 𝑦 = 𝑓(𝑥):

Rules of Differentiation

The tangent line is a straight line; since the derivative is the slope of the tangent line, then the derivative
of a straight line at any point is the slope of the line

In particular,

CALCULUS 1 v.1.0 by JOANNE MICHELLE B. AGUILAR, REE 17


UNIVERSITY OF NUEVA CACERES COLLEGE OF ENGINEERING AND ARCHITECTURE

Based on this, since a constant is a horizontal straight line, then the derivative of a constant is zero:

[8]Rules of Differentiation

The derivative of a constant is 0.

Simple Power Rule:

n is any real

Example 2.1

Constant Multiple Rule:

Example 2.2:

Apply power rule

Example 2.3:

Apply power rule

Sum and Difference Rules:

CALCULUS 1 v.1.0 by JOANNE MICHELLE B. AGUILAR, REE 18


UNIVERSITY OF NUEVA CACERES COLLEGE OF ENGINEERING AND ARCHITECTURE

Example 2.4:

Apply power rule

Product Rule:

In abbreviated form (which may be easier to remember):

Example 2.5:

Differentiate

Step 1: Identify f(x) and g(x)

Step 2: Determine f‘(x) and g’(x)


Apply power rule and Sum
and Difference Rule

Step 3: Substitute the f(x), g(x), f‘(x) and g’(x) into the Product Rule formula, and simplify:

Multiply and distribute

Combine similar terms

CALCULUS 1 v.1.0 by JOANNE MICHELLE B. AGUILAR, REE 19


UNIVERSITY OF NUEVA CACERES COLLEGE OF ENGINEERING AND ARCHITECTURE

Quotient Rule:

In abbreviated form (which may be easier to remember):

Example 2.6:

Differentiate

Step 1: Identify f(x) and g(x)

Step 2: Determine f‘(x) and g’(x)


Apply power rule

Apply power rule

Step 3: Substitute the f(x), g(x), f‘(x) and g’(x) into the Quotient Rule
formula, and simplify:

Distribute to remove the


parenthesis and combine
like terms

CALCULUS 1 v.1.0 by JOANNE MICHELLE B. AGUILAR, REE 20


UNIVERSITY OF NUEVA CACERES COLLEGE OF ENGINEERING AND ARCHITECTURE

[9]Examples 2.7

A. Find the derivative of f ( y)  2 y  4 y  y  33 .


3 2

Solution:

(2𝑦 – 4𝑦 + 𝑦 − 33) = (2𝑦 )– (4𝑦 ) + (𝑦)– (33) Apply derivative of a constant

=2 (𝑦 )– 4 (𝑦 ) + (𝑦) Apply sum and difference rule


= 2(3𝑦 )– 4(2𝑦 ) + 1 Multiply coefficients to simplify
= 6𝑦 − 8𝑦 + 1✓

5
B. Find the derivative of f ( k )  .
k3
Solution:

( ) =5 (𝑘 ) Use the rule for negative exponent


= 5((−3)𝑘 ( )
) Apply power formula
= −15𝑘 ✓ Use the reciprocal rule to make the exponent
= ✓ positive

 1
C. Find the derivative of f (v )  v  v 2  8   .
 v

There are two ways to find the derivative of the function, the product rule and power rule. For now,
we’ll use power rule by first must multiplying out the terms. Recall that v a v b  v ( a  b ) .
1
√𝑣(𝑣 – 8 + ) = 𝑣 𝑣 – 8𝑣 + 𝑣 𝑣
𝑥𝑣
= 𝑣 – 8𝑣 + 𝑣
We can now take the derivative:

𝑑 1 𝑑 𝑑 𝑑
√𝑣 𝑣 – 8 + = 𝑣 –8 𝑣 + 𝑣 Use the rule of exponent in
𝑑𝑣 𝑣 𝑑𝑣 𝑑𝑣 𝑑𝑣
multiplying
5 – 1 – 1 Apply power formula
= 𝑣 –8 𝑣 + − 𝑣
2 2 2
5 1
= 𝑣 – 4𝑣 − 𝑣 ✓
2 2

CALCULUS 1 v.1.0 by JOANNE MICHELLE B. AGUILAR, REE 21


UNIVERSITY OF NUEVA CACERES COLLEGE OF ENGINEERING AND ARCHITECTURE

IV. Learning Activity 2.1


INSTRUCTION: Find the derivative of the given function.

1. f ( x)  2

2 9
2. f ( w)  w
3

3. f (t )  t

1
4. f (g)  3 g 
g

5.  
f (k )  k 2  2k k  2 

h2
6. f ( h) 
h 1

CALCULUS 1 v.1.0 by JOANNE MICHELLE B. AGUILAR, REE 22


UNIVERSITY OF NUEVA CACERES COLLEGE OF ENGINEERING AND ARCHITECTURE

V. Exercises 2.1

INSTRUCTION: Find the derivative of the given function. Use a test booklet for your solution.

1. y  2t 4  10t 2  13t

2. g ( z )  4 z 7  3z 7  9 z

3. h( y )  x  83 x

4 1 8
4. f (t )   3 5
t 6t t

5. 
f ( x)   x  4  2 x  x 2 

y5  5 y3  2 y
6. f ( y) 
y3
3x  1
7. y
2x  3

8. y  3 - 2x 2  3x 

9. y  
x 1 2 x 1 

x2  6
10. y 
2x  7

CALCULUS 1 v.1.0 by JOANNE MICHELLE B. AGUILAR, REE 23


UNIVERSITY OF NUEVA CACERES COLLEGE OF ENGINEERING AND ARCHITECTURE

Week 3: The Derivative and Chain Rule

I. Introduction

[10]In differential calculus, the chain rule is a formula for calculating the derivative of
the composition of two or more functions. That is, if f and g are functions, then the chain rule
expresses the derivative of their composition 𝑓 ∘ 𝑔 (the function which maps x to 𝑓(𝑔(𝑥)) in terms
of the derivatives of f and g.

II. Objectives

After completion of this course, you should be able to:

1. Perform differentiation operations using Chain Rule on algebraic functions.


2. Apply basic algebraic concepts and operations in simplifying derivatives of algebraic
functions.

III. Lecture-Discussion

[8]The Chain Rule is the differentiation of the composition of functions.

Remember that a composition of functions is a function of a function. The output of one function (the
“inner” function) is the input of the other function (the “outer” function), as indicated below.

Figure 3.1 [8]

“The derivative of a composition of functions is equal to the derivative of the outer function with
respect to the inner function (that is, without changing the inner) multiplied by the derivative of the
inner function.”

CALCULUS 1 v.1.0 by JOANNE MICHELLE B. AGUILAR, REE 24


UNIVERSITY OF NUEVA CACERES COLLEGE OF ENGINEERING AND ARCHITECTURE

Example 3.1:
Outer function

Derivative of outside Derivative of inside


Additional Chain Rule Notation:

Example 3.2:

Replace u by x

The General Power Rule:

Example 3.3:

Differentiate

Solution:

Simplify applying rules of


exponents

Apply power rule and


reciprocal theorem

CALCULUS 1 v.1.0 by JOANNE MICHELLE B. AGUILAR, REE 25


UNIVERSITY OF NUEVA CACERES COLLEGE OF ENGINEERING AND ARCHITECTURE

Example 3.4
a) Find the derivative of f ( x )  6 x 2  7 x  4

Solution

 
f ' ( x)  4 6 x 2  7 x 12 x  7 
3

f ' ( x)  412 x  7 6 x  3


2
 7x

b) Find the derivative of y  3 1  8 z


Solution Note: Change radical to exponent before applying the chain rule.
1
y  1  8 z 3
dy 1 2
 1  8 z  3  8

dx 3
dy 8 2
  1  8 z  3

dx 3
3
 5h 4  3 
c) Find the derivative of f ( h)   
  3h  1 
3

Solution Note: Use quotient rule for the derivative of the base.
u  5h 4  3; du  20h3
Let:
v  3h3  1; dv  9h 2
Derivative of the base

d  5h 4  3   3h 3  1 20h 3  5h 4  3  9h 2
 
    
dx   3h 3  1   3h 3  1
2
 
Apply chain rule

f ' h   3 
2

 5h 4  3   3h 3  1 20h 3  5h 4  3  9h 2      Distribute terms to remove
  3h  1 
3
 3h 3  1
2
  the parenthesis and add
like terms
2
 5h 4  3   60h 6  20h 3  45h 6  27h 2
f ' h   3 
  3h  1 
3
 3h 3  1
2
 
f ' h   3
5h  3  15h  20h  27h 
4 2 6 3 2
Factor out h2 and multiply like
 3h  1
3 2
 3h  1 3 2
terms

f ' h   3
5h  3 h  15h  20h  27
4 2 2 4

 3h  1
3
 3h  1
2 3 2

3h 5h  3  15h  20h  27 


2 4 2 4
f ' h  
 3h  1 3 4

CALCULUS 1 v.1.0 by JOANNE MICHELLE B. AGUILAR, REE 26


UNIVERSITY OF NUEVA CACERES COLLEGE OF ENGINEERING AND ARCHITECTURE

d) m  3n  1  3n 2  4  
3

Solution
m  3n  1  3n 2  4 
3
Apply product rule
dm
dn
 
 3n  1 3  3n 2  4  6n    3n 2  4 3
4 3
  Find the GCF and other factors
dm
dn
 4
 
 3  3n 2  4 18n 2  6n   3n 2  4   Simplify the other factor


dm 3 15n 2  6n  4


dn 
 3n 2  4
4

IV. Learning Activity 3.1

INSTRUCTION: Find the derivative of the given function.

1. 
f w  3w 2  5w  2 
8

2. f h   5h 2  3h  1

3. 
f k   k 2  2  5

3
4. f z  
 z 3 2
2

5. f t   2t  1 t 3  t  1
5
 4

CALCULUS 1 v.1.0 by JOANNE MICHELLE B. AGUILAR, REE 27


UNIVERSITY OF NUEVA CACERES COLLEGE OF ENGINEERING AND ARCHITECTURE

V. Exercises 3.1

INSTRUCTION: Find the derivative of the given function. Use a test booklet for your solution.

1. 
g (t )  4t 2  3t  2 
2

 
1
2. g (t )  6 x 2  7 x  2 3

3. h(t )  t 6 5t 2  t

4. V ( z )  z 2 z  8
4 3

5. 𝑓(𝑥) = (𝑥 + 1) (3𝑥– 7)

1
6. f x  
a2  x2

3
 x 1
7. f x    
 x 1

8. y  x 1 x2

4
 x 
9. f x    
1 x 

10. y  3 9 x 2  1

CALCULUS 1 v.1.0 by JOANNE MICHELLE B. AGUILAR, REE 28


UNIVERSITY OF NUEVA CACERES COLLEGE OF ENGINEERING AND ARCHITECTURE

Week 4: Higher Order Derivatives and Implicit Differentiation

I. Introduction

[11]The second derivative, or second-order derivative, is the derivative of a function. The derivative of
the function 𝑓(𝑥) may be denoted by𝑓′(𝑥), and its double (or "second") derivative is denoted by f''(x).
This is read as f double prime of x", or "The second derivative of 𝑓(𝑥)". Since the derivative of function f
is defined as a function signifying the slope of function f, the double derivative is the function signifying
the slope of the first derivative function.

Furthermore, the third derivative is the derivative of the derivative of the derivative of a function, which
can be represented by𝑓′′′(𝑥). This is read as f triple prime of𝑥", or "The third derivative of (x)". This can
continue as long as the resulting derivative is itself differentiable, with the fourth derivative, the fifth
derivative, and so on. Any derivative beyond the first derivative can be referred to as a higher-order
derivative.

In mathematics, an implicit equation is a relation of the form R  x1 ,..., xn  where R is a function of


several variables often a polynomial. For example, the implicit equation of the unit circle is x 2  y 2  1 .

II. Objectives

After completion of this course, you should be able to:

1. Perform differentiation operations on implicit algebraic functions.


2. Apply basic algebraic concepts and operations in simplifying derivatives of algebraic
functions.

III. Lecture-Discussion

[11]Higher-Order Derivatives

Basically, when we speak of derivative, we are referring to the first derivative. Specifically, it is what we
obtained when we differentiate a function once. If we differentiate again, the result is called the second
derivative. Thus, the second derivative is the derivative of the first derivative.

Successive derivatives are named in the same way. For example, the seventh derivative of a function is
obtained by taking derivatives seven times (the derivative of the derivative of the derivative of the
derivative of the derivative of the derivative of the derivative of the function).

CALCULUS 1 v.1.0 by JOANNE MICHELLE B. AGUILAR, REE 29


UNIVERSITY OF NUEVA CACERES COLLEGE OF ENGINEERING AND ARCHITECTURE

[11]Notation

Let f  x  be a function in terms of x. The following are notations for higher order derivatives.

Figure 4.1 [11]

Warning: You should not write f


n
x  to indicate the 𝑛 − 𝑡ℎ derivative, as this is easily confused with
the quantity f  x  all raised to the nth power.
The Leibniz notation, which is useful because of its precision, follows from

d  df  d 2 f
 
dx  dx  dx 2

[8]Examples

Example 4.1

First derivative

Second derivative

Third derivative

Fourth derivative

CALCULUS 1 v.1.0 by JOANNE MICHELLE B. AGUILAR, REE 30


UNIVERSITY OF NUEVA CACERES COLLEGE OF ENGINEERING AND ARCHITECTURE

Example 4.2

2w  1
Find the second derivative of g w  .
3w  2
Solution

23w  2  32 w  1 Apply quotient rule and


g ' w 
3w  22 simplify
7
g ' w   73w  2
2

3w  22

42 Apply chain rule and


g ' ' w  143w  2  3 
3
simplify
3x  23

[12]Implicit Differentiation

A Description of Implicit Differentiation:

To differentiate a function of y with respect to x:

 Differentiate the function as usual when you differentiate x, but when you differentiate y,
multiply the derivative with𝑑𝑦/𝑑𝑥 .

[8]When differentiating keep the following in mind:

 Always differentiate BOTH SIDES of the equation with respect to the same variable.
 The variable that we differentiate with respect to occurs in the denominator of the derivative
expression. For example, if we are seeking𝑑𝑦/𝑑𝑥, then differentiate y with respect to x. If we
are seeking𝑑𝑉/𝑑𝑡, then differentiate V with respect to t.

CALCULUS 1 v.1.0 by JOANNE MICHELLE B. AGUILAR, REE 31


UNIVERSITY OF NUEVA CACERES COLLEGE OF ENGINEERING AND ARCHITECTURE

The following examples were taken from the link [12].

Example 4.3:

Note that we differentiate both sides with respect to x.

Steps:
Differentiate both sides with respect to
x. Apply the sum/difference rule on the
left side.
This term is a function
of x, so regular This term is a function of y, Identify whether the term you differentiate
differentiation. so Implicit differentiation. contains x or y. If it is a function of x, then
regular differentiation. If it is a function of a
variable other than x, like y in this case, then
it is implicit differentiation.

Steps:

Differentiate each term using the applicable rules of


differentiation. Remember, for implicit differentiation,
differentiate as usual, in this case using power
formula, then multiply by dy/dx at the end.

Find dy/dx and simplify.

CALCULUS 1 v.1.0 by JOANNE MICHELLE B. AGUILAR, REE 32


UNIVERSITY OF NUEVA CACERES COLLEGE OF ENGINEERING AND ARCHITECTURE

Example 4.4:

Steps: Differentiate both sides with respect to x. Use


the sum/difference rule in the left side.

To differentiate product of Example 4.4,


use the Product Rule. Be sure to put all x
in one side and dy/dx (left) on the other
side.

One 𝑓𝑥𝑛 in terms of x

One 𝑓𝑥𝑛 in terms of y

Steps:

Note that both parts of the product are


in function of x: 𝑓(𝑥) &𝑔(𝑥).

When doing each differentiation, be sure


to identify whether you need to do
implicit differentiation or regular
differentiation.

Simply apply power formula since f(x) is a function of


x and we differentiate dx.

Implicit differentiation, since f(x) is a function of y and


we differentiate dy.

CALCULUS 1 v.1.0 by JOANNE MICHELLE B. AGUILAR, REE 33


UNIVERSITY OF NUEVA CACERES COLLEGE OF ENGINEERING AND ARCHITECTURE

Steps:
Complete the Product Rule. Be careful in substitution.

Do the same for all products.

Implicit differentiation, since f(x) is a function of y


and we differentiate dy.

Power Rule only since g(x) is a function of x and


we differentiate dx.

Steps:

Substitute all derivatives into the


original equation.

Since we wish to find dy/dx at point (2, –1),


substitute x = 2 & y = –1, then solve for dy/dx.

CALCULUS 1 v.1.0 by JOANNE MICHELLE B. AGUILAR, REE 34


UNIVERSITY OF NUEVA CACERES COLLEGE OF ENGINEERING AND ARCHITECTURE

IV. Learning Activity 4.1


INSTRUCTION: Determine the fourth derivative of the function.

1. f ( w)  3w 7  6 w 4  8w3  12 w  18

2. g (k )  k 3  k 2  k  1

1
3. f (t )  45 t 3   t
8t 2

INSTRUCTION: Find the first derivative of each implicit function.

4. 2 x3  y 2  1  4 y

5. 6 y  xy 2  1

CALCULUS 1 v.1.0 by JOANNE MICHELLE B. AGUILAR, REE 35


UNIVERSITY OF NUEVA CACERES COLLEGE OF ENGINEERING AND ARCHITECTURE

V. Exercises 4.1

INSTRUCTION: Determine the derivative of the function. Use test booklet for your solution.

d2y
1. y  x 2 , find
dx 2
2. f ( x)  4 x 3 , find f ' '
d3y
3. y  4 x, find
dx 3
4. f ( x)  5 x 4 , find f ' ' '
d4y
5. y  5 x , find 4
4

dx
d3y
6. y  3 x 5  2 x, find 3
dx
3 d3y
7. y  2 x  4 x , find 3
3

dx
d3y
8. y   x 2  25 x 2 , find 3
dx
dy
2 x 3  43 xy  1 , find
2
9.
dx
10.  
2
x 2  4 x 2 y 3  1 , find
dy
dx

Week 5: Prelim Exam

CALCULUS 1 v.1.0 by JOANNE MICHELLE B. AGUILAR, REE 36


UNIVERSITY OF NUEVA CACERES COLLEGE OF ENGINEERING AND ARCHITECTURE

Week 6: Derivative of Trigonometric Functions

I. Introduction

[12]The differentiation of trigonometric functions is the mathematical process of finding the derivative of
a trigonometric function, or its rate of change with respect to a variable. Common trigonometric functions
include𝑠𝑖𝑛(𝑥),𝑐𝑜𝑠(𝑥),𝑡𝑎𝑛(𝑥), 𝑐𝑠𝑐(𝑥), 𝑠𝑒𝑐(𝑥)and𝑐𝑜𝑡 (𝑥).

II. Objectives

After completion of this course, you should be able to:

1. Perform differentiation operations on trigonometric functions.


2. Apply trigonometric identities in simplifying derivatives of trigonometric functions.

III. Lecture-Discussion

[13]Table of Derivatives of Trigonometric Functions

The table below summarizes the derivatives of 6 basic trigonometric functions:

Derivative
sin u '  cos udu
cos u '   sin udu
1
tan u '  2
du  sec 2 udu
cos u
cot u '  12   csc 2 udu
sin u
sec u '  sec u tan udu
csc u '   csc u cot udu
Figure 6.1 [13]

CALCULUS 1 v.1.0 by JOANNE MICHELLE B. AGUILAR, REE 37


UNIVERSITY OF NUEVA CACERES COLLEGE OF ENGINEERING AND ARCHITECTURE

Example 6.1

Differentiate the function w  cos 2v  2 sin v

Solution.
w' v   cos 2v  2 sin v '   sin 2v  2v '2 cos v(v)' Use derivative of sine and cosine.
w' v   2 sin 2v  2 cos v Don’t forget to differentiate the angle.

w' v   22 sin v cos v   2 cos v Apply double angle formula


w' v   2 cos v(2 sin v  1) Simplify by factoring out the GCF

Double angle formula: sin 2v  2 sin v cos v

Example 6.2

1 3
Differentiate the function h  tan k  tan k
3
Solution.
The derivative of this function is

dh  1 
  tan k  tan 3 k '
dk  3 

dh 1
 sec2 k  3 tan 2 k tan k ' Use derivative of tangent and chain rule
dk 3
dh
dk
1
 sec2 k  3 tan 2 k sec2 k
3
  Simplify by multiplying coefficients

dh
dk

 sec2 k  tan 2 k sec2 k  Factor out the GCF

dh
dk

 sec2 k 1  tan 2 k  Apply identity 1  tan 2 k  sec 2 x

dh
dk
 
 sec2 k sec2 k  Multiply like terms

dh
 sec4 k
dk

CALCULUS 1 v.1.0 by JOANNE MICHELLE B. AGUILAR, REE 38


UNIVERSITY OF NUEVA CACERES COLLEGE OF ENGINEERING AND ARCHITECTURE

Example 6.3

sin y
Differentiate the function z 
1  cos y
Solution.
By the quotient rule,

 sin y  cos y (1  cos y )  sin y  sin y 


z '  y    '  Distribute and combine like terms
 1  cos y  1  cos y 2
cos y  cos2 y  sin 2 y
z'  y  Apply Pythagorean identity
1  cos y 2
1  cos y
z'  y  Divide common factor
1  cos y 2
1
z'  y 
1  cos y

Pythagorean Identity: sin 2 y  cos2 y  1

Example 6.4

s  cos 2 sin r
Solution.
 
s' r   cos2 sin r '  2 cos sin r  cos sin x ' Applying chain rule
s' r   2 cos sin r   sin sin r sin r '
s' r   2 cos sin r  sin sin r cos r  Differentiate the angle 𝑠𝑖𝑛𝑟, we obtain 𝑐𝑜𝑠𝑟
s' r   2 cos r sin sin r cos sin r Rearrange
let x  sin r Replace the angle to x
s' r    cos r 2 sin x cos x 
Apply sin 2x   2sinxcosx Apply double-angle identity to simplify
s' r   2 cos r sin 2 x Change the angle back x to 𝑠𝑖𝑛𝑟
s' r   2 cos r sin 2 sin r 

CALCULUS 1 v.1.0 by JOANNE MICHELLE B. AGUILAR, REE 39


UNIVERSITY OF NUEVA CACERES COLLEGE OF ENGINEERING AND ARCHITECTURE

IV. Learning Activity 6.1


INSTRUCTION: Determine the derivative of the function.

1. w  sin 2 t

1
2. h  cos
k

3. q  sin 3 r  cos 3 r

4. w  t 2 sin t  2t cos t  2 sin t

y y
5. z  tan  cot
2 2

CALCULUS 1 v.1.0 by JOANNE MICHELLE B. AGUILAR, REE 40


UNIVERSITY OF NUEVA CACERES COLLEGE OF ENGINEERING AND ARCHITECTURE

V. Exercises 6.1

INSTRUCTION: Determine the derivative of the function. Use a test booklet for your solution.

1. y  3 sin x  4 cos x

2. y  x 3 tan x

cos x
3. f ( x) 
1  sin x

4. y  tansin x 

5. y  csc x cot x

sin 2 x
6. y
cos 2 x

7. y  sin2 x   cos2 x

8. g ( x)  sec3 x

9. f ( x)  cos2 x   sin 2 x

sin 3 x 
10. y 
4  5 cos2 x 

CALCULUS 1 v.1.0 by JOANNE MICHELLE B. AGUILAR, REE 41


UNIVERSITY OF NUEVA CACERES COLLEGE OF ENGINEERING AND ARCHITECTURE

Week 7: Derivative of Inverse Trigonometric Functions

I. Introduction

[14]The differentiation of inverse trigonometric functions is the mathematical process of finding the
derivative of an inverse trigonometric function, or its rate of change with respect to a variable. Common
inverse trigonometric functions include sin-1(x), cos-1(x), tan-1(x), csc-1(x), sec-1(x) and cot-1 (x).

II. Objectives

After completion of this course, you should be able to:

1. Compute the derivative of inverse trigonometric functions using basic rules, chain rule,
and implicit differentiation.
2. Apply basic algebraic concepts and trigonometric identities in simplifying derivatives of the
functions.

III. Lecture-Discussion

[14]Derivation of Inverse Trigonometric Functions

Derivative of y  arcsin x

Let y  arcsin x which may be written as x  sin y

We now differentiate both sides of the above with respect to x using the chain rule on the right-hand
side

dy
1  cos y
dx
dy 1

dx cos y
but ,
cos y  1  sin 2 y From Pythagorean identity
Since x  sin y
cos y  1  x 2
hence,
d
arcsin x   1 2
dx 1 x

CALCULUS 1 v.1.0 by JOANNE MICHELLE B. AGUILAR, REE 42


UNIVERSITY OF NUEVA CACERES COLLEGE OF ENGINEERING AND ARCHITECTURE

Derivative of y  arctan x

Let y  arctan x which may be written as x  tan y

We now differentiate both sides of the above with respect to x using the chain rule on the right-hand
side

dy
1  sec 2 y
dx
dy 1

dx sec 2 y
but ,
sec 2 y  1  tan 2 y From Pythagorean identity
sec y  1  x
2 2
Since x  tan y
hence,
d
arctan x   1 2
dx 1 x

Derivative of y  arc sec x

Let y  arc sec x which may be written as x  sec y

We now differentiate both sides of the above with respect to x using the chain rule on the right-hand
side

dy
1  sec y tan y
dx
dy 1

dx sec y tan y
but ,
Since x  sec y and from 1  tan 2 x  sec 2 x this
sec y tan y  x x 2  1
1  tan 2 y  x 2 , tan 2 y  x 2  1, tan y  x2 1

hence,
d
arc sec x   12
dx x x 1

CALCULUS 1 v.1.0 by JOANNE MICHELLE B. AGUILAR, REE 43


UNIVERSITY OF NUEVA CACERES COLLEGE OF ENGINEERING AND ARCHITECTURE

The derivative of Inverse Trigonometric Functions Formulas

Figure 7.1 [14]

Example 7.1

Differentiate w  arcsin 2t .

Solution
Let u  2t ; du  2 and use the for the derivative of inverse sine
dy 2 2
 
dt 1  2t  1  4t 2
2

Example 7.2

Differentiate x  arccos t  3  2

Solution
Let u  t 2  3 ; du  2t and use the formula for the derivative of inverse cosine
2t 2t
x' t    

1 t 2  3 
2

1  t  6t  9
2
 Expand the binomial, distribute and combine
like terms

2t
xt ' 
6t  t 2  8

CALCULUS 1 v.1.0 by JOANNE MICHELLE B. AGUILAR, REE 44


UNIVERSITY OF NUEVA CACERES COLLEGE OF ENGINEERING AND ARCHITECTURE

Example 7.3

3
Differentiate w  sec 1
x
Solution
3
Let u   3 x 1 ; du  3 x 2
x
Apply power formula and use the formula for the derivative of inverse secant
dw  3 x 2  3 x 2
 
dx 3 3
2
3 9
  1 x x2 1 Combine fractions inside the radical
x  x
dw  3x  2

dx 3 9  x 2
Extract the square root of 1/x2 and multiply with x
x x2
3
 2
dw x
 Divide common factor
dx 3
2
9  x2
x
dw 1

dx 9  x2

IV. Learning Activity 7.1

INSTRUCTION: Determine the derivative of the function.

1. y  cot 1 15x

1
2. y  x 2 cot 1 9 x

CALCULUS 1 v.1.0 by JOANNE MICHELLE B. AGUILAR, REE 45


UNIVERSITY OF NUEVA CACERES COLLEGE OF ENGINEERING AND ARCHITECTURE

3. y  x 3 Arc csc 4 x

1
4. y
sin 1 x

5. f ( x )  x tan 1 x

CALCULUS 1 v.1.0 by JOANNE MICHELLE B. AGUILAR, REE 46


UNIVERSITY OF NUEVA CACERES COLLEGE OF ENGINEERING AND ARCHITECTURE

V. Exercises 7.1

INSTRUCTION: Determine the derivative of the function. Use test booklet for your solution.

1
1. y  Arc tan
x

2. y  Arc sin x  1

x 1
3. y  Arc tan
x 1

1 x
4. y Arc tan
a a

5. y  Arc coscos x 

f ( x )  7 x  3 cos 1 2 x
12
6.

7. f ( x)  sin 3x tan 1 6 x

 x4
8. f ( x )  Arc cot  
 x5

2x
9. f ( x) 
Arc sec 4 x


10. y  Arc cos x  3
2

CALCULUS 1 v.1.0 by JOANNE MICHELLE B. AGUILAR, REE 47


UNIVERSITY OF NUEVA CACERES COLLEGE OF ENGINEERING AND ARCHITECTURE

Week 8: Derivative of Logarithmic Functions

I. Introduction

[15]Logarithmic functions are the inverses of exponential functions. The inverse of the
exponential function y = ax is x = ay. The logarithmic function y = logax is defined to be equivalent to the
exponential equation x = ay. ... It is called the logarithmic function with base a.

II. Objectives

After completion of this course, you should be able to:

1. Compute the derivative of a logarithmic function using basic rules, chain rule, and implicit
differentiation.
2. Apply basic algebraic concepts, trigonometric identities, and properties of logarithm in
simplifying derivatives of the functions.

III. Lecture-Discussion

[16]Natural Log (𝑳𝒏)

The natural logarithm of a number is its logarithm to the base of the mathematical constant e, where e
is an irrational and transcendental number approximately equal to 2.718281828459. The natural
logarithm of x is generally written as 𝑙𝑛 𝑥, loge x, or sometimes, if the base e is implicit, simply log x.

The natural logarithm function, if considered as a real-valued function of a real variable, is the inverse
function of the exponential function, leading to the identities:

e ln x  x if x  0
ln e x  x
Like all logarithms, the natural logarithm maps multiplication into addition:

ln xy  ln x  ln y

CALCULUS 1 v.1.0 by JOANNE MICHELLE B. AGUILAR, REE 48


UNIVERSITY OF NUEVA CACERES COLLEGE OF ENGINEERING AND ARCHITECTURE

[17]Derivative of 𝒍𝒏(𝒖) and 𝐥𝐨𝐠 𝒂 𝒖

Using the Chain Rule, we get

d 1 du
1 ln u  
dx u dx
d 1 du
2 log a u  
dx u ln a dx
Properties of Natural Logarithm

1. ln 1  0
2. ln e  1
3. ln e x  x

4. ln y  x ln y
x

5. ln xy   ln x  ln y
x
6. ln   ln x  ln y
 y
Source: [18]

[19]Examples

Example 8.1

Find the derivative of w  ln r 2

Use the Power Rule for Logarithm:

w  ln r 2  2 ln r
Then simply multiply 2 times the derivative of 𝑙𝑛 r using the log derivative formula (1).

w'  2 ln r '
2
w' r  
r

Example 8.2


Find the derivative of d  2 ln 3 f  1
2

We put

u  3 f 2 1

CALCULUS 1 v.1.0 by JOANNE MICHELLE B. AGUILAR, REE 49


UNIVERSITY OF NUEVA CACERES COLLEGE OF ENGINEERING AND ARCHITECTURE

Then the derivative of u is given by

du
u'  6f
df

So the final answer using formula (1) is:

du
Apply the derivative of 𝑙𝑛𝑢 formula
df
d ' f   2
u
6f
d ' f   2 2 Multiply the coefficients
3 f 1
12 f
d ' f  
3 f 2 1

Example 8.3

Find the derivative of j  ln 1  2 k  .


3

First, we apply Power Rule of Logarithm to simplify:

log a n  n log a

You can write

j  ln 1  2 k   3 ln 1  2k 
3

Then we put

u  1 2k
So

du
u'   2
dk
So our answer using formula (1) is:

du
dj
 3 dk
dk u
dj 2
3
dk 1  2k
dj 6

dk 1  2k

CALCULUS 1 v.1.0 by JOANNE MICHELLE B. AGUILAR, REE 50


UNIVERSITY OF NUEVA CACERES COLLEGE OF ENGINEERING AND ARCHITECTURE

Example 8.4

 
Find the derivative of z  3 log 7 y 2  1 .

We put

u  y2 1

giving

du
u'   2y
dy

Applying the derivative of logarithm formula (2), we have:

dz 1
3 2 2y
dy 
y  1 ln 7 
dz 6y
 2
dy 
y  1 ln 7
dz y
 3.083 2
dy y 1 
The value 3.083 comes from simplified form of coefficients.

Example 8.5

Find the derivative 𝑑𝑚/𝑑𝑛 of 3 ln nm  sin m  n 2 .

Observe that this is an implicit function. We can simplify it first by applying product rule of logarithm:

3 ln nm  sin m  n 2

3ln n  ln m  sin m  n 2

This makes the equation easier to differentiate. Now, taking derivatives:

 1 1 dm  dm
3    cos m  2n
 n m dn  dn
Gathering terms gives us:

CALCULUS 1 v.1.0 by JOANNE MICHELLE B. AGUILAR, REE 51


UNIVERSITY OF NUEVA CACERES COLLEGE OF ENGINEERING AND ARCHITECTURE

3 dm dm 3
 cos m  2n  Factor out 𝑑𝑚/𝑑𝑛
m dn dn n
dm  3  3 Solve for 𝑑𝑚/𝑑𝑛
  cos m   2n 
dn  m  n
 3
 2n  mn
dm n
  Simplify complex fraction by multiplying 𝑚𝑛 to both
dn  3  numerator and denominator
  cos m mn
m 
2n 2 m  3m
m' n  
3n  mn cos m

IV. Learning Activity 8.1 [20]

INSTRUCTION: Determine the derivative of the function.

1. q  lnsin r 

3 3
2. w  log 3   
t t

3. 
z  ln y  y 2  a 2 

 1 k 
4. h  ln 

 1  k 

5. w
 
log 2 t 2
t2

CALCULUS 1 v.1.0 by JOANNE MICHELLE B. AGUILAR, REE 52


UNIVERSITY OF NUEVA CACERES COLLEGE OF ENGINEERING AND ARCHITECTURE

V. Exercises 8.1

INSTRUCTION: Determine the derivative of the function. Use test booklet for your solution.

1. y  ln ln 2 x 4

2. y  cos ln 4 x 3

5
 4x 4 
3. y  ln 3 
 x 3

4. 
y  log 3 3 x 5  5 
5

5. 
y  log 5  5 x 5  2 
5

6. y  log 5 tan 4 x 4

7. 
y  ln 2 x 3  x 
2

8. 
y  ln cos x 2 
9. y  x ln 3 x

10. y  sin x 
x

Week 9: Midterm Exam

CALCULUS 1 v.1.0 by JOANNE MICHELLE B. AGUILAR, REE 53


UNIVERSITY OF NUEVA CACERES COLLEGE OF ENGINEERING AND ARCHITECTURE

Week 10: Derivative of Exponential Functions

I. Introduction

Exponential Function is a function whose base is a constant or e raised to an argument/variable exponent.

II. Objectives

After completion of this course, you should be able to:

1. Compute the derivative of exponential functions using basic rules, chain rule, and implicit
differentiation.
2. Apply basic algebraic concepts, trigonometric identities and properties of logarithm in
simplifying derivatives of the functions.

III. Lecture-Discussion

[21] The function f(x) = 2x is called an exponential function because the exponent x is the variable. Do
not confuse it with the function g(x) = x2, in which the variable is the base x. The table shows the
derivatives of exponential functions. Scroll down the page for more examples and solutions on how to
use the derivatives of exponential functions formula.

Derivatives of Exponential Functions


d u du d u du
1 e  eu 2 a  a u ln a
du dx du dx
Table 10.1 [21]

In general, an exponential function is of the form f(x) = ax where a is a positive constant.

The derivative of the Natural Exponential Function

The exponential function f(x) = ex has the property that it is its own derivative. This means that the slope
of a tangent line to the curve y = ex at any point is equal to the y-coordinate of the point.

We can combine the above formula with the chain rule to get

(1)

CALCULUS 1 v.1.0 by JOANNE MICHELLE B. AGUILAR, REE 54


UNIVERSITY OF NUEVA CACERES COLLEGE OF ENGINEERING AND ARCHITECTURE

[21]Examples

Example 10.1:

Differentiate the function i = e

Solution:

Apply formula (1) for natural exponential function

di d
 e sin j sin j 
dj dx
di
 e sin j cos j
dj

Example 10.2:

Differentiate the function z = e–

Solution:

Using the Product Rule and formula (1), we get

let u  e -3y du  3e -3y


v  sin4y dv  4cos4y


dz 3 y
dy
  
e sin 4 y  e 3 y cos 4 y 4   sin 4 y e 3 y  3

dz
 4e 3 y cos 4 y  3e 3 y sin 4 y
dy Factor out the GCF
dz
 e 3 y 4 cos 4 y  3 sin 4 y 
dy

Derivative of 𝒂𝒖

d u
dx
 
a  a u ln a
du
dx
(2)

CALCULUS 1 v.1.0 by JOANNE MICHELLE B. AGUILAR, REE 55


UNIVERSITY OF NUEVA CACERES COLLEGE OF ENGINEERING AND ARCHITECTURE

Example 10.3:

Differentiate 𝑦 = 𝑥 + 3

Solution:

y' 
d 3 x
dx
 
x  3  3 x 2  3 x ln 3 Apply power formula to the first term and formula (2)

Example 10.4:

Differentiate 𝑦 = 5

Solution:

y' 
dx

d 2 x 1
5 
 52 x 1 ln 52 Use formula (2)

y '  52 x 5ln 5 2 Breakdown 5 and multiply 5 with 2


y '  10ln 55 2 x

IV. Learning Activity 10.1

INSTRUCTION: Determine the derivative of the function.

1. w  103t

2
2. q  et

3. h  sin e3k  

ln 2t
4. w
e 2t  2

5. q  cos 2t et   2
1

CALCULUS 1 v.1.0 by JOANNE MICHELLE B. AGUILAR, REE 56


UNIVERSITY OF NUEVA CACERES COLLEGE OF ENGINEERING AND ARCHITECTURE

V. Exercises 10.1

INSTRUCTION: Determine the derivative of the function. Use a test booklet for your solution.

ex 1
1. y
ex  1

2. 
y  ex x2  2 x  2 
3. y  x2 2x

2
4. y  43 x

1
5. y3 x

2
6. y  10 x

7. y  e x sin x

8. f ( x)  etan 2 x

2
ex
9. y
x

4
10. y  33 cos x

CALCULUS 1 v.1.0 by JOANNE MICHELLE B. AGUILAR, REE 57


UNIVERSITY OF NUEVA CACERES COLLEGE OF ENGINEERING AND ARCHITECTURE

Week 11: Derivative of Hyperbolic and Inverse Hyperbolic Functions

I. Introduction

[22]In mathematics, hyperbolic functions are equivalents of the usual trigonometric functions defined
for the hyperbola rather than on the circle: just as the points (cos t, sin t) form a circle with a unit radius,
the points (cosh t, sinh t) form the right half of the equilateral hyperbola.

II. Objectives

After completion of this course, you should be able to:

1. Compute the derivative of hyperbolic and inverse hyperbolic functions using basic rules,
chain rule, and implicit differentiation.
2. Apply basic algebraic concepts, hyperbolic identities and properties of logarithm in
simplifying derivatives of the functions

III. Lecture-Discussion

[22]Exponential Functions Definition

Hyperbolic sine: the odd part of the exponential function, that is

Hyperbolic cosine: the even part of the exponential function, that is

Hyperbolic tangent:

Hyperbolic cotangent: for x ≠ 0,

Hyperbolic secant:

CALCULUS 1 v.1.0 by JOANNE MICHELLE B. AGUILAR, REE 58


UNIVERSITY OF NUEVA CACERES COLLEGE OF ENGINEERING AND ARCHITECTURE

Hyperbolic cosecant: for x ≠ 0,

[22]Hyperbolic Function Identities

Odd and Even Functions

[22]Hyperbolic sine and cosine satisfy:

the last of which is similar to the Pythagorean trigonometric identity. One also has

[22]Sums of arguments

Particularly

[22]Subtraction formulas

CALCULUS 1 v.1.0 by JOANNE MICHELLE B. AGUILAR, REE 59


UNIVERSITY OF NUEVA CACERES COLLEGE OF ENGINEERING AND ARCHITECTURE

[22]Inverse Functions as Logarithms

[22]Derivatives

CALCULUS 1 v.1.0 by JOANNE MICHELLE B. AGUILAR, REE 60


UNIVERSITY OF NUEVA CACERES COLLEGE OF ENGINEERING AND ARCHITECTURE

[23]Examples (Use the formulas from the list above and the chain rule)

Example 11.1

Differentiate the function y  tanh x 2  3 x  


Solution

dy d

dx dx
    
tanh x 2  3 x  sec h 2 x 2  3 x 2 x  3  Apply Chain Rule

Example 11.2

1
Differentiate the function y 
sinh x 2
Solution

dy d  1  d
    sinh x  2
2 
Simplify by getting the reciprocal
dx dx  sinh x   dx
 2sinh x  cosh x
3
Apply Chain Rule and derivative of sinh

Example 11.3

 x
Differentiate the function y  sinh 1  
3
Solution
d   x  1
y' x   sinh 1    
dx   3  x2
3 1 Simplify radicand by adding terms
9
1
y' x  
9  x2 Extract the square root of 1/9
3
9
1
y' x  
3
9  x2 Divide common factor
3
1
y' x  
9  x2

CALCULUS 1 v.1.0 by JOANNE MICHELLE B. AGUILAR, REE 61


UNIVERSITY OF NUEVA CACERES COLLEGE OF ENGINEERING AND ARCHITECTURE

Example 11.4

Differentiate the function w  arcsin htan x 

Solution

dw 1
 tan x ' Use the formula from the list
dx tan 2 x  1
dw
dx

1
sec x 
2
1
tan x  1
2
Apply identities for 1  tan 2 x  sec 2 x and sec x 
cos x
dw 1 1
 2
dx tan 2 x  1 cos x
dw 1 1
 2
dx sec 2 x cos x
dw 1 1
 2
dx 1 cos x
cos 2 x
dw 1 1

dx 1 cos 2 x
cos x
dw 1
  sec x
dx cos x

IV. Learning Activity 11.1

INSTRUCTION: Determine the derivative of the function.

1.  
w  tanh t 2

2. q  r sinh r  cosh r

CALCULUS 1 v.1.0 by JOANNE MICHELLE B. AGUILAR, REE 62


UNIVERSITY OF NUEVA CACERES COLLEGE OF ENGINEERING AND ARCHITECTURE

3. w  sinh 2 t

4. q  arctan hcos t 

5. w  arctan htanh r 

CALCULUS 1 v.1.0 by JOANNE MICHELLE B. AGUILAR, REE 63


UNIVERSITY OF NUEVA CACERES COLLEGE OF ENGINEERING AND ARCHITECTURE

V. Exercises 11.1

INSTRUCTION: Determine the derivative of the function. Use a test booklet for your solution.

1. y  arctan hsin x 

2. y  cosh x

3. f ( x)  tan(4 x)

4. f ( x)  sinh x tanh x

5. f ( x)  x 2 sinh 1 2 x 

6. f ( x)  tanh 1 ( x )

7. f ( x)  sec h 1 ( 1  x )

a
8. y  arccos h
x

9. y  csc h 2 3x

10. y  sec h ln x
2

CALCULUS 1 v.1.0 by JOANNE MICHELLE B. AGUILAR, REE 64


UNIVERSITY OF NUEVA CACERES COLLEGE OF ENGINEERING AND ARCHITECTURE

Week 12: Polynomial Curves

I. Introduction

[24]A polynomial function is a function such as a quadratic, a cubic, a quartic, and so on, containing only
non-negative integer powers of x. We can give a general definition of a polynomial, and define its
degree.

A polynomial is a function of the form

f x   an x n  an1 x n1  ...  a2 x 2  a1 x  a0

The degree of a polynomial is the highest power of x in its expression. Constant (non-zero) polynomials,
linear polynomials, quadratics, cubic and quartic are polynomials of degree 0, 1, 2, 3 and 4 respectively.
The function 𝑓(𝑥) = 0 is also a polynomial, but we say that its degree is ‘undefined’.

[25]This unit explains how differentiation can be used to compute the equations of the tangent and
normal to a curve. The tangent is a straight line which just touches the curve at a given point. The
normal is a straight line which is perpendicular to the tangent. To determine the equations of these lines
we shall make use of the fact that the equation of a straight line passing through the point with
coordinates (x1, y1) and having gradient m is given by

y  y1
m
x  x1

We also make use of the fact that if two lines with gradients m 1 and m2 respectively are perpendicular,
then 𝑚1𝑚2 = −1. In order to master the techniques explained here it is vital that you undertake
plenty of practice exercises so that they become second nature.

II. Objectives

After completion of this course, you should be able to:

1. Use derivative to find the slope of the curve.


2. Determine the equation of the tangent line to the curve and the normal line.

CALCULUS 1 v.1.0 by JOANNE MICHELLE B. AGUILAR, REE 65


UNIVERSITY OF NUEVA CACERES COLLEGE OF ENGINEERING AND ARCHITECTURE

III. Lecture-Discussion

[25]Consider a function 𝑓(𝑥) such as that shown in Figure 10.1. When we calculate the derivative, f ′, of
the function at a point 𝑥 = 𝑎 say, we are finding the gradient of the tangent to the graph of that
function at that point. Figure 10.1 shows the tangent drawn at 𝑥 = 𝑎. The gradient of this tangent is
𝑓 ′ (𝑎).
[26]At a given point on a curve, the gradient of the curve is equal to the gradient of the tangent to the
curve.

Figure 12.1The tangent drawn at x = a has gradient 𝑓 ′ (𝑎) [25]

Figure 12.2The Tangent and Normal Line [26]

We will use this information to calculate the equation of the tangent to a curve at a particular point, and
then the equation of the normal to a curve at a point.

[26]The derivative (or gradient function) describes the gradient of a curve at any point on the curve.
Similarly, it also describes the gradient of a tangent to a curve at any point on the curve.

[26]To determine the equation of a tangent to a curve:


1. Find the derivative using the rules of differentiation.
2. Substitute the x-coordinate of the given point into the derivative to calculate the gradient of the
tangent.
3. Substitute the gradient of the tangent and the coordinates of the given point into an
appropriate form of the straight line equation.
4. Make y the subject of the formula.

The normal to a curve is the line perpendicular to the tangent to the curve at a given point.

CALCULUS 1 v.1.0 by JOANNE MICHELLE B. AGUILAR, REE 66


UNIVERSITY OF NUEVA CACERES COLLEGE OF ENGINEERING AND ARCHITECTURE

[27]Examples

Example 12.1

Find the equation of the tangent to the curve y  x at the point (1, 1).

Solution

Figure 12.3 Graph of y  x


Step 1: Find the first derivative

  d  2
1
dy d
 x   x 
 Change radical to exponent
dx dx dx  
1
dy 1 2 1 Apply power formula
 x
dx 2
dy 1 Take the reciprocal and change
 exponent to radical
dx 2 x

Step 2: Using point (1, 1), let 𝑥 = 1, 𝑦 = 1 and substitute 𝑥 = 1 to the first derivative
1
f '  x0   f ' 1 
2

Step 3: Using these three values


1
𝑥 = 1, 𝑦 = 1 and f '  x0  
2

Substitute into the point-slope form equation of the line:


y  y0  f '  x0  x  x0 

This yields the equation of the tangent line:


1
y 1  x  1
2
2 y  2  x 1
x  2 y 1  0
Answer

CALCULUS 1 v.1.0 by JOANNE MICHELLE B. AGUILAR, REE 67


UNIVERSITY OF NUEVA CACERES COLLEGE OF ENGINEERING AND ARCHITECTURE

Example 12.2
Find a point on the curve y  w  2 w  3 at which the tangent is parallel to the x - axis.
2

Solution
Since the tangent is parallel to the x−axis, the derivative is equal to zero at this point. Hence,
dy

dw dw
d

w2  2w  3  2w  2
We find that 0  2w  2
w 1
Then solve for y by substituting 𝑤 = 1
y  w2  2 w  3
y  12  21  3
y  4
Thus, the point on the curve at which the tangent line is parallel to x – axis is (1, -4).

Example 12.3

Find the equation of the normal line to the curve y  t  e at t0 = 0.


3 t

Solution
Determine the value of the y0 at 𝑡 = 0.
y0  03  e0  1

The derivative is given by


dy d 3 t

dt dt

x e 
dy
 3t 2  et
dt

At the point x0 = 0, it equals


dy
 3  02  e0  1
dt

Thus, the equation of the normal is written as follows:


1
y  y0  x  x0  Note: The slope of the normal line is
f ' x0  the negative reciprocal of the slope of
1 the tangent line (slopes of two lines
y 1    x  0 that are perpendicular)
1
y 1   x
x  y 1  0

CALCULUS 1 v.1.0 by JOANNE MICHELLE B. AGUILAR, REE 68


UNIVERSITY OF NUEVA CACERES COLLEGE OF ENGINEERING AND ARCHITECTURE

Example 12.4

The equation of the tangent line to the graph of a function is 2 x  y  4  0 at 𝑥 = 1. Find the
equation of the normal line passing through the same point.

Solution.
We rewrite the equation of the tangent in slope intercept form as y  2 x  4 and determine the slope
and y−coordinate of the point of tangency.

The slope of the tangent line is −2.


Find y0 using point of tangency at 𝑥 = 1: y0  21  4  2

Then the slope of the normal line is the negative reciprocal of the slope of the tangent line, we get that
the slope of the normal is equal to 1/2. So the equation of the normal can be written as
y  y0  m x  x0 
y  y0  f '  x0  x  x0 
1
y2  x  1
2
2 y  4  x 1
x  2y  3  0
Answer

IV. Learning Activity 12.1

Problem Solving

1. Find the equation of the tangent line to the curve 𝑦 = 𝑥 at 𝑥 = 1.

CALCULUS 1 v.1.0 by JOANNE MICHELLE B. AGUILAR, REE 69


UNIVERSITY OF NUEVA CACERES COLLEGE OF ENGINEERING AND ARCHITECTURE

x 1
2. Find the equation of the normal to the graph of the function y  at the point where x = 2.
x 1

3. Find the equations of the tangent line and normal line to the parabola y=2x 2 at the point (2, 8).

4. Write equations of the tangent line to the graph of the function y  x x  1 at x=2.

5. Find the equation of the normal line to the curve x  y  2 x  6  0 at the point (−1, 3).
3 2

CALCULUS 1 v.1.0 by JOANNE MICHELLE B. AGUILAR, REE 70


UNIVERSITY OF NUEVA CACERES COLLEGE OF ENGINEERING AND ARCHITECTURE

V. Exercises 12.1

1. Determine the equation of the normal line to the curve 𝑦 = arccot at 𝑡 = 1.


2. As shown in figure 12.4, a tangent line is drawn to the function 𝑦 = at the point (1, 1).
Compute the length of the tangent line segment AB in the first quadrant.

Figure 12.4

3. As shown in figure 12.5, the tangent and normal lines are drawn to the parabola 𝑦 = 𝑥 at the
point 𝑥 = 2. Determine the length of the line segment AB between the points of intersection of
the lines with the x−axis.

Figure 12.5

4. Refer to figure 12.6, compute the area of the triangle formed by the tangent line to the function
𝑦 = 3 − 𝑥 drawn at the point (1, 2) and the coordinate axes.

Figure 12.6

5. Determine the equation of the tangent line drawn to the curve 𝑦 − 4𝑥 − 6𝑥𝑦 = 0 at the
point of tangency P (1, 2).

Week 13: Semi-final Exam

CALCULUS 1 v.1.0 by JOANNE MICHELLE B. AGUILAR, REE 71


UNIVERSITY OF NUEVA CACERES COLLEGE OF ENGINEERING AND ARCHITECTURE

Week 14: Increasing and Decreasing Functions

I. Introduction

Before we can describe how functions behave, we can identify intervals over which the function is
changing either increasing or decreasing in definite ways. We can say that if the interval of the function
is increasing then it is going up and if it is decreasing it is going down. Similarly, when the graph is going
up, its slope is positive. While when the graph is going down, the slope is negative. Since slope and
derivative are the same, we can tell whether the function is increasing or decreasing with the derivative
of a function.

II. Objectives

After completion of this course, you should be able to:

1. Use the first derivative test to find the relative maxima and relative minima of the function.
2. Apply the second derivative tests to solve relative extrema and concavity of the function.

III. Lecture-Discussion

One way to determine whether the function is increasing, decreasing or, constant on any intervals in its
domain is by graphing. An easier way is by using the derivative of the function. If > 0, then the
function is increasing on the interval, and if < 0, then the function is decreasing on the interval.
The behavior of the curve on any interval whether it is increasing or decreasing is known as
monotonicity of the function.

Figure 14.1 Monotonicity of the Curve

Table 14.1 Monotonicity of Figure 14.1

CALCULUS 1 v.1.0 by JOANNE MICHELLE B. AGUILAR, REE 72


UNIVERSITY OF NUEVA CACERES COLLEGE OF ENGINEERING AND ARCHITECTURE

The First Derivative Test

A function 𝑓(𝑥) is continuous on an open interval containing critical values. If 𝑓(𝑥) is differentiable on
the interval, except probably at the critical values, then 𝑓(𝑥) at the critical values can be classified as
follows:
a) Relative/local Maximum (maximum point) if 𝑓(𝑥) changes from positive to negative at m.
b) Relative/local Minimum (minimum point) if 𝑓(𝑥) changes from negative to positive at n.
c) If there is no sign change, then the critical value is not a relative minimum or maximum.

Figure 14.2 Curve with Critical Values within Intervals

Table 14.2 Relative Extrema of Figure 14.2

Note: First derivative is positive if the graph is increasing and negative if it is decreasing.

The Second Derivative Test

The second derivative test is used to determine the concavity of the function. Concavity refers to the
curvature of the curve. The point on the graph where the concavity changes is the point of inflection
(poi). It can be determined by setting the second derivative equal to zero(𝑓 (𝑥) = 0).

Figure 14.3 Point of Inflection Table 14.3 Concavity of Figure 14.3

CALCULUS 1 v.1.0 by JOANNE MICHELLE B. AGUILAR, REE 73


UNIVERSITY OF NUEVA CACERES COLLEGE OF ENGINEERING AND ARCHITECTURE

The graph of a twice-differentiable function 𝑦 = 𝑓 (𝑥) is:


1. Concave up on any interval where 𝑓’’(𝑥) > 0, and
2. Concave down on any interval where 𝑓’’(𝑥) < 0.

Figure 14.4 Critical Points Table 14.4 Concavity and Extrema of Figure 14.4

Note: Second derivative is positive if the graph opens upward and negative if it opens downward.

Example 14.1
Find the maximum and minimum point of the function y  x  6 x  9 using the first derivative test.
3 2

Solution
Step 1: Find the first derivative of the function y  x  6 x  9
3 2

y'  3x 2  12x
Step 2: Set the first derivative to zero and solve for x (critical numbers).
y '  3x 2  12 x
0  3x 2  12 x Factor out the GCF
0  3 x x  4  Set the factors to zero
x  0,x  4

Step 3: Use number line to identify intervals by plotting the critical numbers. Choose a value of x in
every interval and substitute it to the first derivative y '  3 x 2  12 x .

y + - +

0 4

y '  1  3 1  12 1  15


2
Positive (15) when x = -1 first interval
y ' 1  31  121  9
2
Negative (-9) when x = 1, middle
interval
y ' 5  35  125  15
2
Positive (15) when x = 5, last interval

According to the result of the first derivative test, (a) from positive to negative at zero, thus x = 0 is
maximum, and (b) from negative to positive at 4, thus x = 4 is minimum.

CALCULUS 1 v.1.0 by JOANNE MICHELLE B. AGUILAR, REE 74


UNIVERSITY OF NUEVA CACERES COLLEGE OF ENGINEERING AND ARCHITECTURE

Step 4: Solve for y using the function y  x  6 x  9


3 2

y  0 3  60  9  9
2
When x = 0
y  4  64  9  23 When x = 4
3 2

Step 5: Therefore the maximum point occurs at P (0, 9) and minimum point occurs at P (4, -23).

Example 14.2
Find the maximum and minimum point of the function y  x  6 x  9 and point of inflection using the
3 2

second derivative test.

Solution
Step 1: Find the first derivative of the function y  x  6 x  9
3 2

y'  3x 2  12x
Step 2: Set the first derivative to zero and solve for x (critical numbers).
y '  3x 2  12 x
0  3x 2  12 x Factor out the GCF
0  3 x x  4  Set the factors to zero
x  0,x  4
Step 3: Solve the second derivative of the function y  x  6 x  9
3 2

y'  3x 2  12x
y ' '  6 x  12
Step 4: Substitute the critical numbers to the second derivative.

y ' ' 0  60  12  12 Y’’ is negative thus the curve opens downward
y' ' 4  64  12  12 Y’’ is positive thus the curve opens upward

Step 5: Compute for y using the function y  x  6 x  9


3 2

y  0 3  60  9  9
2
When x = 0
y  4  64  9  23 When x = 4
3 2

Step 6: According to Table 14.4, when the concavity of the curve is downward, the relative extrema is
maximum and if the curve opens upward the relative extrema is minimum. Therefore the maximum
point occurs at P (0, 9) and minimum point occurs at P (4, -23).

Step 7: Point of Inflection


Let the second derivative equal to zero and solve for x
y ' '  6 x  12

CALCULUS 1 v.1.0 by JOANNE MICHELLE B. AGUILAR, REE 75


UNIVERSITY OF NUEVA CACERES COLLEGE OF ENGINEERING AND ARCHITECTURE

0  6 x  12
x2
Find y by substituting x = 2 to the equation y  x  6 x  9
3 2

y  2  62  9  7
3 2

Thus the point of inflection is at P (2, -7).

IV. Learning Activity 14.1

Problem Solving

1. Given the function 𝑓(𝑡) = 2𝑡 + 3𝑡 – 12𝑡 + 7 determine the critical numbers.

2. Make a number line to identify intervals by plotting the critical numbers obtained from
problem#1 of the function 𝑓(𝑡) = 2𝑡 + 3𝑡 – 12𝑡 + 7. Choose a value of x in every
interval and substitute it to the first derivative. Identify the increasing and decreasing intervals.

3. Solve for the maximum and minimum point of the function 𝑓(𝑡) = 2𝑡 + 3𝑡 – 12𝑡 + 7
using the data gathered from problems 1 and 2.

4. Determine the concavity of the curve at each critical number of the function 𝑓(𝑡) = 2𝑡 +
3𝑡 – 12𝑡 + 7.

5. Find the point of inflection of the function 𝑓(𝑡) = 2𝑡 + 3𝑡 – 12𝑡 + 7.

CALCULUS 1 v.1.0 by JOANNE MICHELLE B. AGUILAR, REE 76


UNIVERSITY OF NUEVA CACERES COLLEGE OF ENGINEERING AND ARCHITECTURE

V. Exercises 14.1

Problem Solving

1. Given the function f t   2t 3  3t 2  12t  1 determine the critical numbers.


2. Make a number line to identify intervals by plotting the critical numbers obtained from
problem#1 of the function f t   2t 3  3t 2  12t  1 . Choose a value of x in every interval and
substitute it to the first derivative. Identify the increasing and decreasing intervals.
3. Solve for the maximum and minimum point of the function f t   2t 3  3t 2  12t  1 using the
data gathered from problems 1 and 2.
4. Determine the concavity of the curve at each critical number of the function
f t   2t 3  3t 2  12t  1 .
5. Find the point of inflection of the function f t   2t 3  3t 2  12t  1 .

CALCULUS 1 v.1.0 by JOANNE MICHELLE B. AGUILAR, REE 77


UNIVERSITY OF NUEVA CACERES COLLEGE OF ENGINEERING AND ARCHITECTURE

Week 15: Related Rates

I. Introduction

Related rates problems are about the relationships of at least two changing quantities and how the rate
of change of one affects the rate of change of another. Usually, it is a problem in which we know one of
the rates of change at a given instant.

II. Objectives

After completion of this course, you should be able to:

1. Create an equation (or several equations) relating the variables defined.


2. Use implicit differentiation in solving rate of change problems.

III. Lecture-Discussion

[28]Here are the important steps in planning and solving related rates problems

1. Sketch a figure or else create a mathematical model of the situation.


2. Determine the constant and variable quantities in your figure.
3. Classify the independent variable in the problem. This is typically time but it need not be.
4. Identify which rates are given and which rate you need to find.
5. Apply the mathematical model to write down an equation among the variable quantities.
6. Differentiate this equation with respect to the independent variable quantity, typically making
use of implicit differentiation.
7. Compute the quantity asked.

Examples [29]

Problem 15.1 [29]

A man placed a ladder 20 ft. in length against a wall. On the floor, the foot of the ladder starts to slide
away from the wall at the rate of 1 , how fast is the top of the ladder sliding down the wall when the
foot of the ladder is 12 feet from the wall?

dy dy
1
Given: dt Find: dt when 𝑥 = 12

Figure 15.1 Triangle

CALCULUS 1 v.1.0 by JOANNE MICHELLE B. AGUILAR, REE 78


UNIVERSITY OF NUEVA CACERES COLLEGE OF ENGINEERING AND ARCHITECTURE

Solution

x 2  y 2  20 2 Differentiate with respect to time


dx dy Divide both sides by two to simplify
2x  2y 0
dt dt
dx dy
x y 0
dt dt

Solving for y when 𝑥 = 12: Thus

x 2  y 2  20 2 dy
121  16 0 Substitute x and y
12 2  y 2  20 2 dt
dy
144  y 2  400 16  12 Solve for 𝑑𝑦/𝑑𝑡
dt
y 2  256 dy 3
y  16 
dt 4

The negative derivative means that distance y is decreasing. Therefore the rate at which the top of
ladder is sliding down is ¾ .

Problem 15.2 [29]

Given a rectangle with dimensions that are continuously changing. The measurement of the width
increases at the rate of 3 while the length decreases at the rate of 2 . At an instant, the rectangle
is a 20-inch square. How fast is its area changing 3 seconds later?

dx
3
dt
dA
dy
Given:  2 Find dt 3 seconds later
dt
Figure 15.2 Rectangle

CALCULUS 1 v.1.0 by JOANNE MICHELLE B. AGUILAR, REE 79


UNIVERSITY OF NUEVA CACERES COLLEGE OF ENGINEERING AND ARCHITECTURE

Solution ‘x’ and ‘y’ 3 seconds later


A  xy x  20  33  29
dA dy dx y  20  3 2  14
x y
dt dt dt
dA
 29 2   143  16 3 seconds later
dt

Since is negative, the area is decreasing at the rate of 16 inches/sec.

Problem 15.3 [29]

A long and narrow container full with water is 2m in length and has a cross section in the shape of an
isosceles trapezoid 30 cm wide at the bottom, 60 cm wide at the top, and a height of 50 cm. If the
container leaks water at the rate of 2000 cm 3/min, how fast is the water level falling when the water is
20 cm deep?

Given Find

dV dh
 2000  ? when ℎ = 20
dt dt
Note: Volume of the water is 𝑉 = 𝐴 𝑙

where A = area of the trapezoid and I = length of the trough

The area of the trapezoid is (


, a and b are the base
)

Figure 15.3

h
A x  30 Area of the trapezoid based from the
dimensions of figure 15.3
2

CALCULUS 1 v.1.0 by JOANNE MICHELLE B. AGUILAR, REE 80


UNIVERSITY OF NUEVA CACERES COLLEGE OF ENGINEERING AND ARCHITECTURE

Solution

V  Al
h
V l x  30 When 𝑙 = 200
2
V  100hx  30

Figure 15.4 Triangle ABC is similar to triangle ADE since their


corresponding sides are proportional

By similar triangle: From figure 15.4 x  2 y  30 eqn. 2

y h

15 50
3
y h eqn. 1
10
Substitute eqn. 1 to eqn. 2

3 
x  2 h   30
 10 
3
x  h  30
5
Substitute x to the volume equation:

V  100h x  30 
3 
V  100h h  60 
5 
V  60h  6000h
2

dV dh
 120h  6000  When substitute = −2000 and ℎ = 20
dt dt
dh
 2000  8400
dt
dh 2000

dt 8400
dh 5
  cm/min
dt 21

The negative sign indicate that the water level is decreasing at a rate of cm/min

CALCULUS 1 v.1.0 by JOANNE MICHELLE B. AGUILAR, REE 81


UNIVERSITY OF NUEVA CACERES COLLEGE OF ENGINEERING AND ARCHITECTURE

IV. Learning Activity 15.1 [30]

Problem Solving

1. A conical tank 20 feet high and a radius of 5 feet is used to store water. How fast is the water
level rising if the water is being pumped into the tank at the rate of 100 ft 3/min when the water
height is 10 feet?

2. A water tank in the shape of an inverted cone is 8 feet high and 4 feet radius. is being emptied
at the rate of 2 cubic feet per second. The height of the cone is 8 feet and the radius is 4 feet.
Find the rate of change of the water level if the tank is being emptied at the rate of 2 ft 3/sec
when the depth is 6 feet.

3. Ben 6 feet tall walks away at a rate from the light at the top of the 16 feet pole. Determine
the rate at which the tip of his shadow is moving when he is 20 feet from the pole.

4. The volume of a cube is growing at a rate of 2 in3/min. Determine the rate at which the surface
of the cube is growing when the side is 8 inches.

CALCULUS 1 v.1.0 by JOANNE MICHELLE B. AGUILAR, REE 82


UNIVERSITY OF NUEVA CACERES COLLEGE OF ENGINEERING AND ARCHITECTURE

V. Exercises 15.1

Problem Solving

1. A ladder 10 feet long is standing against the side of a house. The foot of the ladder is moved
away from the side of the house at the rate of 1 ft/sec. Determine the rate at which the top of
the ladder is falling down the side of the house 1 second after the foot begins being pulled away
from the house after 8 seconds. Ans. -1.333 ft/sec

2. Determine the rate of change of the radius of a sphere if the volume is increasing at the rate of
8π ft3/sec when the radius is 6 feet. Ans. 1/8 ft/sec

3. The area of the rectangle is changing with the width of a rectangle that is increasing at a rate of
2 cm/sec, while the length is increasing at 3 cm/sec. Find the rate at which the area is increasing
when the width is 4 cm and the length is 5 cm. Ans. 22 cm2/sec

4. Two trains leave from the same station at the same time. Train X travels due North at 30 mph,
while train Y travels due East at 40 mph. Find the rate at which the distance between the cars
changing after 1 hour. Ans. 50 mph

5. The rate at which the spherical red balloon is inflated is 100 cm 3 /sec. Determine the rate at
which the radius is changing when the diameter of the balloon is 50 cm? Ans. 1/25π

6. Car A and B are traveling, car A is going due East at a rate of 90 km/hr and car B is going due
South at a rate of 60 km/hr. The two cars are traveling toward the intersection of the two roads.
Determine the rate at which the cars approaching each other at the instant when car A is 0.2 km
and car B is 0.15 km from the intersection. Ans. 108 km/hr

7. A big yellow balloon is rising at a rate of 20 feet per second. It is 10 feet above the ground at the
time that the back end of a blue car is directly below the bottom of the balloon. The blue car is
traveling at 40 feet per second. Determine the rate of change at which the distance between the
bottom of the balloon and the point on the ground directly below the back of the blue car one
second after the back of the car is directly below the balloon? Ans. 44 ft/sec

CALCULUS 1 v.1.0 by JOANNE MICHELLE B. AGUILAR, REE 83


UNIVERSITY OF NUEVA CACERES COLLEGE OF ENGINEERING AND ARCHITECTURE

Week 16: Applied Maxima and Minima

I. Introduction

Optimization is one of the most essential applications of differential calculus. When we are looking for
the best way to perform a given task whether looking for the maximum volume to construct from a
given material or the minimum/least cost of material required to create an object, we used
optimization.

II. Objectives

After completion of this course, you should be able to:

1. Solve optimization problems using differentiation.


2. Apply differential calculus to solve real world problems.

III. Lecture-Discussion

Here are the important steps in planning and solving optimization problems

1. Sketch a figure or else create a mathematical model of the situation.


2. Determine the constant and variable quantities in your figure.
3. Create an equation signifying the quantity to be maximized or minimized. This variable quantity
will normally be characterized in terms of two or more variables.
4. Determine any constraints or relations between the variables to eliminate all but one
independent variable. This changes the equation obtained in step 3 into a function defined by
two variables. Identify the domain of this function appropriate to the problem.
5. (a) When the function is continuous on a closed interval, apply the closed interval method to
calculate its absolute maximum or minimum values.
(b) But when there is only one critical value within the interval, apply either the first or second
derivative test to calculate whether it is a relative maximum or relative minimum. The computed
value will be the absolute maximum value or absolute minimum value, respectively.

CALCULUS 1 v.1.0 by JOANNE MICHELLE B. AGUILAR, REE 84


UNIVERSITY OF NUEVA CACERES COLLEGE OF ENGINEERING AND ARCHITECTURE

Example 16.1 [29]


Calculate the maximum and minimum combined area of a square and/or a rectangle whose length is
three times its width, formed from a 24-inch long piece of wire.

Given
Let: x = side of a square
y = width of the rectangle

Figure 16.1

4 x  8 y  24 The sum of the perimeter of the


x  2y  6 square and rectangle is equal to the
length of the wire

Solution
For the critical number
A  x2  3y2 Total area in terms of x and y

A y   6  2 y   3 y 2 Total area in terms of y only


2

A'  y   26  2 y  2   6 y Apply chain rule


A'  y   24  8 y  6 y Distribute and combine like terms
A'  y   24  14 y
0  24  14 y
Set the first derivative to zero
24 12
y  Solve for y (critical number)
14 7

Compute 𝐴(𝑦) at the critical number and at the endpoints of the interval.

(a) When 𝑦 = 0 (all the wire is used to form a square)

A y   6  2 y   3 y 2
2

A0  36

CALCULUS 1 v.1.0 by JOANNE MICHELLE B. AGUILAR, REE 85


UNIVERSITY OF NUEVA CACERES COLLEGE OF ENGINEERING AND ARCHITECTURE

(b) When 𝑦 = , the critical number

2 2
 12   24   12 
A    6    3 
7  7  7
2 2
 12   18   12 
A      3 
7 7 7
 12  108
A    15.43
7 7

(c) When all wire is used to form a rectangle (𝑥 = 0)


when x  0
x  2y  6 A y   6  2 y   3 y 2
2

2y  6 A3  27
y3

Therefore:
The maximum area happens when all the wire is used to form a square and the minimum area occurs
when 𝑦 = at critical number.

Example 16.2 [29]


A 24-inch by 15-inch piece of sheet metal is used to create an open box by cutting squares of equal size
from the corners and folding up the sides. Find the dimension of the cutout that maximizes the volume
of the box.

Given
Let: x = dimension of the cutout

Figure 16.2

Dimensions of the box in terms of x


l  24  2 x
w  15  2 x
hx

CALCULUS 1 v.1.0 by JOANNE MICHELLE B. AGUILAR, REE 86


UNIVERSITY OF NUEVA CACERES COLLEGE OF ENGINEERING AND ARCHITECTURE

Solution
Volume of the box
V  lwh
V  x   24  2 x 15  2 x  x  Volume in terms of x

V  x   4 x 3  78 x 2  360 x Multiply and combine like terms

V '  x   12 x 2  156 x  360 Solve for the first derivative



V '  x   12 x 2  13 x  30  Factor out the GCF and the other factor

V '  x   12x  10  x  3 Use trial and error to factor out trinomials


x  10; x  3 Set the factors to zero and solve for x

There are two values of x. When 𝑥 = 10, the volume is negative, thus the value was rejected. When
𝑥 = 3 the volume is positive. Use 2nd derivative test to ensure that 𝑥 = 3 is relative maximum.

V '  x   12 x 2  156 x  360 Solve for the second derivative


V ' '  x   24 x  156 And substitute x = 3
when x  3
When the second derivative is negative it means that the curve
V ' ' 3  243  156  84 opens downward and the relative extreme is maximum (refer to
2nd derivative test).

Thus, maximum volume of 486 in3 occurs when 𝑥 = 3.

Example 16.3 [29]


Determine the dimensions of the rectangle of largest area that can be inscribed in an equilateral triangle
whose side measures 20 units.

Given
Let: w = width of the rectangle and y = length of the rectangle
w = 2x

Figure 16.3

CALCULUS 1 v.1.0 by JOANNE MICHELLE B. AGUILAR, REE 87


UNIVERSITY OF NUEVA CACERES COLLEGE OF ENGINEERING AND ARCHITECTURE

Find h (refer to figure 16.3)


h 2  10 2  20 2 Apply Pythagorean Theorem and solve for h
h 2  10 2  400
h 2  300
h  300  10 3
Solution
By similar triangle:
y 10  x

10 3 10
 10  x  By similar triangle solve for y
y  10 3  
 10 
y  3 10  x  Equation 1

Area of the rectangle = 2𝑥𝑦


A x   2 x 3 10  x  Express the area in terms of x (use eqn.1)


A x   2 3 10 x  x 2 
A' x  2 3 10  2 x  Solve the first derivative

0  2 3 10  2 x  Set the first derivative to zero


2 x  10 Solve for x
x5

Second derivative test


A' x  2 3 10  2 x  When the second derivative is negative it means that the curve
A' ' x  2 3 0  2 opens downward and the relative extreme is maximum (refer to
2nd derivative test).
A' ' x  4 3

Solve for y (use eqn.1)


y  3 10  x 
y  3 10  5
y5 3

Thus, the dimensions of the rectangle inscribed in an equilateral triangle are 𝑤 = 2𝑥 = 10 (width)
and y  5 3 (length).

CALCULUS 1 v.1.0 by JOANNE MICHELLE B. AGUILAR, REE 88


UNIVERSITY OF NUEVA CACERES COLLEGE OF ENGINEERING AND ARCHITECTURE

IV. Learning Activity 16.1 [31]

1. Determine the two nonnegative numbers whose totality is 9 and the product of first number
and the square of the second number is a maximum.

2. A rectangular enclosure with three parallel partitions using 500 feet of fencing. Determine the
dimensions that will maximize the total area of the enclosure.

3. An open rectangular box with square base is to be created from 48 square feet of material. Find
the dimensions of the open rectangular box that will maximizes the possible volume.

CALCULUS 1 v.1.0 by JOANNE MICHELLE B. AGUILAR, REE 89


UNIVERSITY OF NUEVA CACERES COLLEGE OF ENGINEERING AND ARCHITECTURE

4. An open top container in the form of a right circular cylinder has surface area 3𝜋 square feet.
Determine the height and base radius that will maximize the volume of the open top container.

V. Exercises 16.1 [32]

1. Determine the dimensions of a most economical cylindrical can (r and h) that can hold up to
20𝜋 cubic meters of water when the material for the top and bottom costs P10/m 2 and material
for the side costs P8/m2. Ans. 2 m, 5 m

2. Determine the largest area of a rectangle with a base on the x-axis and upper vertices on the
parabola 𝑦 = 12 – 𝑥 . Ans. 32 sq. unit

3. A rectangular enclosure with a divider in the middle is to be constructed using 120 meters of
fencing. What are the dimensions that should be used in order to maximize the bounded area?
Ans. 30 m, 20 m

4. A closed box is to be created from a piece of paper with size of 8 cm by 15 cm by cutting four
squares on each corners and folding up the sides. What is the dimension of the square that
maximizes the volume of the box? Ans. 5/3 cm

5. An open top rectangular box with a square base and a volume of 216 cubic inches is to be
created. Determine the minimum surface area of the box. Ans. Ans. 108√4in2

Week 17: Summative Test/Review Week

Week 18: Final Exam

CALCULUS 1 v.1.0 by JOANNE MICHELLE B. AGUILAR, REE 90


UNIVERSITY OF NUEVA CACERES COLLEGE OF ENGINEERING AND ARCHITECTURE

Works Cited

[1] [Online]. Available: https://byjus.com/maths/relations-and-functions/.

[2] [Online]. Available: https://www.slideshare.net/yayang0130/relations-and-functions-66540774.

[3] [Online]. Available:


http://www.montereyinstitute.org/courses/DevelopmentalMath/COURSE_TEXT2_RESOURCE/U17_L2_T3_t
ext_final.html.

[4] [Online]. Available: https://www.slideshare.net/njit-ronbrown/lecture-4-the-limit-of-a-function.

[5] [Online]. Available: https://www.powershow.com/search/presentations/ppt/derivative.

[6] [Online]. Available:


https://www.math.ucdavis.edu/~kouba/CalcOneDIRECTORY/defderdirectory/DefDer.html#PROBLEM%201.

[7] [Online]. Available: https://en.wikipedia.org/wiki/Derivative.

[8] [Online]. Available: https://www.slideserve.com/abrial/section-2-3-the-product-and-quotient-rules-and-


higher-order-derivatives.

[9] [Online]. Available: https://www.matheno.com/calculus-1/calculating-derivatives-problems-solutions/.

[10] [Online]. Available: https://en.wikipedia.org/wiki/Chain_rule.

[11] [Online]. Available: https://en.wikibooks.org/wiki/Calculus/Higher_Order_Derivatives.

[12] [Online]. Available: https://en.wikipedia.org/wiki/Differentiation_of_trigonometric_functions.

[13] [Online]. Available: https://www.math24.net/derivatives-trigonometric-functions.

[14] [Online]. Available:


https://www.analyzemath.com/calculus/Differentiation/derivatives_of_inverse_trigonometric_functions.ht
ml.

[15] [Online]. Available: https://www.mathwarehouse.com/calculus/derivatives/how-to-differentiate-with-


logarithmic-functions.php.

[16] [Online]. Available:


https://en.wikipedia.org/wiki/Natural_logarithm#:~:text=The%20natural%20logarithm%20of%20a,is%20im
plicit%2C%20simply%20log%20x..

[17] [Online]. Available: https://www.onlinemathlearning.com/derivative-ln.html.

[18] [Online]. Available:


https://www.math.ucdavis.edu/~kouba/CalcOneDIRECTORY/logdiffdirectory/LogDiff.html.

CALCULUS 1 v.1.0 by JOANNE MICHELLE B. AGUILAR, REE 91


UNIVERSITY OF NUEVA CACERES COLLEGE OF ENGINEERING AND ARCHITECTURE

[19] [Online]. Available: https://www.intmath.com/differentiation-transcendental/5-derivative-logarithm.php.

[20] [Online]. Available: https://www.math24.net/derivatives-logarithmic-functions/.

[21] [Online]. Available: https://www.onlinemathlearning.com/exponential-derivative.html.

[22] [Online]. Available: https://en.wikipedia.org/wiki/Hyperbolic_functions.

[23] [Online]. Available:


https://math.libretexts.org/Courses/Monroe_Community_College/MTH_211_Calculus_II/Chapter_6%3A_A
pplications_of_Integration/6.9%3A_Calculus_of_the_Hyperbolic_Functions.

[24] [Online]. Available: http://www.mathcentre.ac.uk/resources/uploaded/mc-ty-polynomial-2009-1.pdf.

[25] [Online]. Available: http://www.mathcentre.ac.uk/resources/uploaded/mc-ty-tannorm-2009-1.pdf.

[26] [Online]. Available: https://www.siyavula.com/read/maths/grade-12/differential-calculus/06-differential-


calculus-04.

[27] [Online]. Available: https://www.math24.net/tangent-normal-lines.

[28] [Online]. Available: https://oregonstate.edu/instruct/mth251/cq/Stage9/Lesson/relatedRates.html.

[29] E. D. a. B. Don, "How to Solve Word Problems in Calculus," in How to Solve Word Problems in Calculus,
McGraw-Hill.

[30] [Online]. Available: https://www.math24.net/related-rates/.

[31] [Online]. Available:


https://www.math.ucdavis.edu/~kouba/CalcOneDIRECTORY/maxminsol2directory/MaxMinSol2.html#SOLU
TION%208.

[32] [Online]. Available: https://web.auburn.edu/holmerr/1617/Textbook/optimization-print.pdf.

CALCULUS 1 v.1.0 by JOANNE MICHELLE B. AGUILAR, REE 92


UNIVERSITY OF NUEVA CACERES COLLEGE OF ENGINEERING AND ARCHITECTURE

APPENDIX

I. The solution to Learning Activities

 Learning Activity 1.1


1.

f '  x   lim
4  
xh3  4 x3 
h 0 h
x3  xh3 x3  xh3
f '  x   lim
h 0 h x3  xh3
f '  x   lim
 x  3    x  h  3
h 0 h  x3 xh3 
h
f '  x   lim
h 0 h  x3 xh3 
 1 
f '  x   lim 
h 0
 x  3  x  h  3 
1
f ' x   
x3  x03
1
f ' x   
2 x3

2.

x  h  1  x  1
2  x  h  2  x
f ' x   lim
h0 h
1  x  h   1 x  1 
f ' x   lim  
h0 h 2   x  h 
 2  x 
1  x  h  12  x   x  12  x  h 
f ' x   lim  
h0 h
 2  x  h 2  x  
1 3h 
f ' x   lim 
h0 h 2  x  h 2  x  
 
3
f ' x   lim
h0 2  x  h 2  x 

3
f ' x  
2  x  02  x 
3
f ' x  
2  x 2

CALCULUS 1 v.1.0 by JOANNE MICHELLE B. AGUILAR, REE 93


UNIVERSITY OF NUEVA CACERES COLLEGE OF ENGINEERING AND ARCHITECTURE

3.

2 2

f ' x   lim
x  h  3  x 3
h 0 h

x  h    x 
1 1
2 3 2 3
f ' x   lim
h 0 h

x  h    x  x  h    x  h  x   x 


1 1 2 1 1 2
2 3 2 3 2 3 2 3 2 3 2 3
f ' x   lim
x  h    x  h  x   x 
2 1 1 2
h 0 h 2 3 2 3 2 3 2 3

f ' x   lim
x  h 2  x 2

   x  h  x   x  
h 0 2 1 1 2
h x  h 
2 3 2 3 2 3 2 3

 
x  2 xh  h  x
2 2 2
f ' x   lim

   x  h  x   x  
h 0 2 1 1 2
h x  h 
2 3 2 3 2 3 2 3

 
2 xh  h 2
f ' x   lim

   x  h  x   x  
h 0 2 1 1 2
h x  h 
2 3 2 3 2 3 2 3

 
2x  h
f ' x   lim
x  h    x  h   x   x 
h 0 2 1 1 2
2 3 2 3 2 3 2 3

2x  0
f ' x  
x  0   x  0  x   x 
2 1 1 2
2 3 2 3 2 3 2 3

2x
f ' x  
x   x  x   x 
2 1 1 2
2 3 2 3 2 3 2 3

2x
f ' x   4 4 4
x  x3  x3
3

2x
f ' x   4
3x 3

2
f ' x   1
3x 3

CALCULUS 1 v.1.0 by JOANNE MICHELLE B. AGUILAR, REE 94


UNIVERSITY OF NUEVA CACERES COLLEGE OF ENGINEERING AND ARCHITECTURE

4.

cos 3 x  h   cos 3 x
f '  x   lim
h0 h
cos3 x  3h   cos 3 x
f '  x   lim
h0 h
cos 3x cos 3h  sin 3x sin 3h  cos 3x
f '  x   lim
h0 h
cos 3xcos 3h  1  sin 3 x sin 3h
f '  x   lim Use squeeze theorem
h0 h
cos 3h  1
 cos 3xcos 3h  1 sin 3 x sin 3h  f '  x   lim 0
f '  x   lim    h 0 3h
h0
 h h 
sin 3h
 3 cos 3 xcos 3h  1 3 sin 3 x sin 3h  f '  x   lim 1
f '  x   lim   
h 0 3h
h0
 3h 3h 
f '  x   lim 3 sin 3 x 
h0

f '  x   3 sin 3x

5.

x  h   1  x  1
f ' x   lim
x  h 2  3x  h  x 2  3x
h0 h
  
1  x  h  1 x 2  3x  x  1 x 2  2 xh  h 2  3 x  3h 
f ' x   lim 

h0 h
 
x 2  2 xh  h 2  3 x  3h x 2  3x 
  
1   x 2 h  xh 2  2 xh  h 2  3h 
f ' x   lim  2
h0 h 
 x  2 xh  h  3x  3h x  3x 
2 2   
1
f ' x   lim  2

h  x 2  xh  2 x  h  3  
h0 h 
 x  2 xh  h  3x  3h x  3x 
2 2   
 x 2  xh  2 x  h  3
f ' x   lim 2

h0 x  2 xh  h 2  3 x  3h x 2  3 x  
 x  2x  3 2
f ' x  
x  3x x 2  3x
2
 
x  2x  3
2
f ' x   
x 2
 3x 
2

CALCULUS 1 v.1.0 by JOANNE MICHELLE B. AGUILAR, REE 95


UNIVERSITY OF NUEVA CACERES COLLEGE OF ENGINEERING AND ARCHITECTURE

 Learning Activity 2.1


1.

f ' x   0

2.

d 2 9 2
f ' w    w   9 w  6 x
9 1 8

dw  3  3

3.

 t   dtd  t  1  12
1
d 1
f ' t   2  t 
 2
dt   2 t

4.

d  3 1  d  3  
1 1 2 3
1  1 
f ' g   g  g  g 2  g 3  g 2
dg  g  dg   3
 2

5.

f ' (k ) 
d
dk
    
k 2  2k k  2  k 2  2k  k  22k  2  k 2  2k  2k 2  2k  4k  4

f ' (k )  3k 2  8k  4

6.

d  h  2  h  1  h  2  3
f ' ( h)    
dh  h  1  h  1 2
h  12

 Learning Activity 3.1

1. 
f ' ( w)  8 3w 2  5w  2  6w  5
7

2.

 
1
f ' (h)  5h 2  3h  1 2

 
1
1
5h 2  3h  1 2 10h  3

f ' (h) 
2
10h  3
f 'h 
2 5h 2  3h  1

CALCULUS 1 v.1.0 by JOANNE MICHELLE B. AGUILAR, REE 96


UNIVERSITY OF NUEVA CACERES COLLEGE OF ENGINEERING AND ARCHITECTURE

3.


f ' k   5k k 2  3 2k  
4


f ' k   10k k 2  3  4

4.

f ' z  
d 
 
3
2
 d


dz  z  3  dz
2

 3

3 z 2  3  6 z 2  3 2 x   
2

12 x
z2  3
3
 
 

5.

f ' t  
d
dt
       
2t  15 t 3  t  1 4  2t  15 4 t 3  t  1 3 3t 2  1  t 3  t  1 4 52t  14 2

f ' t   22t  1 t 3  t  1 22t  13t  1  5t  t  1
4 3 2 3

f ' t   22t  1 t  t  1 26t  2t  3t 1  5t  5t  5


4 3 3 3 2 3

f ' t   22t  1 t  t  1 12t  4t  6t 2  5t  5t  5


4 3 3 3 2 3

f ' t   22t  1 t  t  1 17t  6t 9t  3


4 3 3 3 2

 Learning Activity 4.1


1.

f ' w  21w6  24 w3  24 w 2  12


f ' ' w   126 w5  72 w 2  48w
f ' ' ' w   630 w 4  144 w  48
f 4 w   2520 w3  144

2.

g ' (k )  3k 2  2k  1
g ' ' ( k )  6k  2
g ' ' ' (k )  6
g 4 k   0

CALCULUS 1 v.1.0 by JOANNE MICHELLE B. AGUILAR, REE 97


UNIVERSITY OF NUEVA CACERES COLLEGE OF ENGINEERING AND ARCHITECTURE

3.

1
f (t )  4 5 t 3   t
8t 2
3 1
1
f t   4t 5  t  2  t 2
8
3 1
3 1 1 1 1
f ' t   4 t 5  2 t 3  t 2
5 8 2
2
12  5 1 3 1  12
f ' t   t  t  t
5 4 2
7
24  3 1 3
f ' ' t    t 5  t  4  t 2
25 4 4
12 5
168  5 3 2
f ' ' ' t   5
t  3t  t
125 4
17 7
2016  5 15 
f 4 t    t  15t  6  t 2
625 16

4.
2w3  y 2  1  4 y 3w
y' 
6 w 2  2 yy '  4 y ' y2

y ' 2 y  4  6 w 2  3w 
3 y  6  3w 
6w  y  2 3  3wy '  y  2 
y'   y' '  
2y  4  y  22  y  2 2
6w y' ' 
3 y  6 y  2  3w3w 3 y 2  6 y  6 y  12  3w 2

y'  
2 y  2   y  23  y  23
y' 
3w
y' '  

3 y 2  12 y  12  3w 2 3 y 2  6 y  6  w 2 
y2  y  23  y  23

5.
6 y  xy 2  1
6 y ' x2 yy'  y 2  0
y ' 2 xy  6   y 2
y2
y' 
2 xy  6

CALCULUS 1 v.1.0 by JOANNE MICHELLE B. AGUILAR, REE 98


UNIVERSITY OF NUEVA CACERES COLLEGE OF ENGINEERING AND ARCHITECTURE

y2
y' 
2 xy  6

y' ' 
2 xy  62 yy'  y 2 2 xy'2 y   4 xy 2 y'12 yy'2 xy 2 y'2 y 3
2 xy  62 2 xy  62
y2
2 y  xy  6   2 y 3
y' '  
 
2 xy 2 y '12 yy'2 y 3 y ' 2 xy 2  12 y  2 y 3 2 xy  6

2 xy  62 2 xy  62 2 xy  62
2 y 3 xy  6 
 2 y3
2 xy  6 2 y 3 xy  6   2 y 3 2 xy  6  2 y 3 xy  6  2 xy  6  2 xy 4
y' '     
2 xy  62 2 xy  63 2 xy  63 2 xy  63

 Learning Activity 6.1


1.

dw  1 1 
 2 sin t cos t  t 2 
dt 2 
dw  1  Use double-angle formula/identity to
 2 sin t cos t  
dt 2 t  simplify
dw
dt
   1 
 sin 2 t  
2 t 
dw sin 2 t 

dt 2 t
2.

dh
dk
1
1
  sin  k  2  2 sin
k k
1
k
3.

dq
 3 sin 2 r cos r  3 cos 2 r  sin r 
dr
dq
 3 sin 2 r cos r  3 sin r cos2 r  3 sin r cos r sin r  cos r 
dr
dq  2  3
 3  sin r cos r sin r  cos r   sin 2r sin r  cos r 
dr 2 2

CALCULUS 1 v.1.0 by JOANNE MICHELLE B. AGUILAR, REE 99


UNIVERSITY OF NUEVA CACERES COLLEGE OF ENGINEERING AND ARCHITECTURE

4.

dw d 2

dt dt
 
t sin t  2t cos t  2 sin t  t 2 cos t  sin t 2t    2t sin t  2 cos t   2 cos t

dw 2
 t cos t  2t sin t  2t sin t  2 cos t  2 cos t  t 2 cos t
dt
5.

 y y
2 sin 2  cos2 
dz 1 2 y 1 2 y 1 1 2 2 21
 sec  csc     
dy 2 2 2 2 2 cos2 y y y y  y y
2
2 sin 2 4 sin 2 cos2  2 sin cos 
2 2 2 2  2 2
dz 2
 2  2 csc2 y
dy sin y

 Learning Activity 7.1


1.

1
f ' ( x) 
1  15 x 
2

1
 15
1  225 x 2
15

1  225 x 2
2.
1 1
1 2 1
f ' ( x)  x Arc tan 9 x  x 2 9
1  9 x 
2
2
1 1
1  9
 x 2 Arc tan 9 x  x 2
2 1  81x 2
1
1   18 x 
 x 2  Arc tan 9 x  
2  1  81x 2 
1  18 x 
 1 
Arc tan 9 x  
 1  81x 2 
2x 2

1  18 x 
  Arc tan 9 x  
2 x 1  81x 2 

CALCULUS 1 v.1.0 by JOANNE MICHELLE B. AGUILAR, REE 100


UNIVERSITY OF NUEVA CACERES COLLEGE OF ENGINEERING AND ARCHITECTURE

3.

4
f ' ( x)  3 x 2 Arc csc 4 x  x 3
4x 4 x 2  1
4x 3
 3 x 2 Arc csc 4 x 
4 x 16 x 2  1
x3
 3 x 2 Arc csc 4 x 
x 16 x 2  1

4.
dy d

dx dx
sin 1 x
1


  sin 1 x 
2 d

dx

sin 1 x 
1

 2

sin 1 x 1  x 2
5.
1
1 1 2
f  x   tan 1
xx x
1  x2
2
x
 tan 1 x 
21  x 

 Learning Activity 8.1


1.

dq d 1
 lnsin r   cos r  cot r
dr dr sin r

2.

dw d   3 3  1  1  1 3t  1   1  3 t 
 log 3      3  2   3  2     2   3 2    2   1
dt dt   t t   3 ln 3  t   t  3 ln 3  t   t  t  3 ln 3 
t
dw 3  t  3 ln 3  t  3 ln 3
 2  2
dt t  3 ln 3  t ln 3

CALCULUS 1 v.1.0 by JOANNE MICHELLE B. AGUILAR, REE 101


UNIVERSITY OF NUEVA CACERES COLLEGE OF ENGINEERING AND ARCHITECTURE

3.

dz d

dy dy

ln y  y 2  a 2  
1
y  y2  a2

1  2y
 2 y2  a2

 1
 y  y2  a2
 y2  a2 

 y2  a2
y 

   
dz y2  a2  y 1
dy

 
y  y2  a2 y2  a2

y2  a2

4.

dh d   1  k  d  1  1  k  1 1  1  k  1  1  k  
 ln   ln    
dk dh   1  k  dh  2  1  k  2 1  k  1  k 2 
1 k
dh 1  1  k   1  k  1  k  1  k  2  1 1 1
       2
dk 2  1  k  1  k   21  k 1  k  1  k 1  k  k  1k  1 k  1
2

5.

 
2t 3
dw d  log 2 t 2  t 2 ln 2
 2t log 2 t 2   
2t 3  2tt 2 ln 2 log 2 t 2   
2t 3 1  ln 2 log 2 t 2
    
dt dt  t 2  t4 t 4t 2 ln 2 t 4t 2 ln 2
   
dw 2 1  log 2 t 2 ln 2

dt t 3 ln 2

 Learning Activity 10.1


1.

dw
dt
 
 3 ln 10 103t

2.

dq 2
 2tet
dt

CALCULUS 1 v.1.0 by JOANNE MICHELLE B. AGUILAR, REE 102


UNIVERSITY OF NUEVA CACERES COLLEGE OF ENGINEERING AND ARCHITECTURE

3.

dh
dk
  
 cos e3k 3e3k

dh
dk
 3e3k cos e3k  

4.

dw d  ln 2t 
e 12t

t
 ln 2t 2e 
2 2t

  
dt dt  e  2 
2t
e  2 2t 2

dw e  2   t ln 2t 2e  e  2   2te
2t 2t 2t 2t
ln 2t
 
t e  2  t e  2 
2 2
dt 2t 2t

5.

dq d

dt dt
        
cos 2t et 1  cos 2t et 1 2t   et 1  2 sin 2t   et 1 cos 2t 2t   2 sin 2t 
2 2 2 2

dq
dt
 
 2 et 1 t cos 2t  sin 2t 
2

 Learning Activity 11.1

1.

dw d

dt dt
   
tanh t 2  sec h 2t 2 2t 
2t

cosh 2 t 2

2.

dq d
 r sinh r  cosh r   r cosh r  sinh  sinh r  r cosh r
dr dr

CALCULUS 1 v.1.0 by JOANNE MICHELLE B. AGUILAR, REE 103


UNIVERSITY OF NUEVA CACERES COLLEGE OF ENGINEERING AND ARCHITECTURE

3.

dw d

dt dt
 
sinh 2 t  2 sinh t cosh t  sinh 2t

4.

dq d 1
 arctan hcosh t    sin t    sin2t   1   csc t
dt dt 1  cos t
2
sin t sin t

5.
dw d 1 1
 arctan htanh r   sec h 2t 
dr dr 1  tanh t
2
 sinh 2 t 
1  2 
 cosh 2 t
 cosh t 
dw 1 1
   sec ht
dr  cosh t  sinh t 
2 2
cosh 2t
 2
 cosh t
2

 cosh t 

 Learning Activity 12.1


1.

dy dy
 31  3
2
 3x 2 at x1 = 1
dx dx

For y: y1  1  1
3

dy
y  y1  x  x1 
dx
y  1  3 x  1
3x  y  2  0

2.

CALCULUS 1 v.1.0 by JOANNE MICHELLE B. AGUILAR, REE 104


UNIVERSITY OF NUEVA CACERES COLLEGE OF ENGINEERING AND ARCHITECTURE

dy x  1   x  1 2
 
dx x  12
x  12
At point x1  2
2 1
y1  3
2 1
dy 2
  2
dx 2  12
So the equation of the normal line is
1
y  y1  
dy
x  x1 
dx
1
y 3 x  2
2
x  2y  4  0

3.

dy dy
 4 x at x1  2 8
dx dx
egn of tangent line is
dy
y  y1  x  x1 
dx
y  8  8 x  2 
8x  y  8  0

So the equation of the normal line is


1
y  y1  
dy
x  x1 
dx
1
y 8   x  2
8
x  8 y  66  0

CALCULUS 1 v.1.0 by JOANNE MICHELLE B. AGUILAR, REE 105


UNIVERSITY OF NUEVA CACERES COLLEGE OF ENGINEERING AND ARCHITECTURE

4.

dy x 2 x-1  x 3 x  2
 x 1   
dx 2 x-1 2 x-1 2 x-1
dy 32  2
at x1  2  2
dx 2 2-1
y1  2
egn of tangent line is
dy
y  y1  x  x1 
dx
y  2  2 x  2
2x  y  2  0

5.

3 x 2  2 yy '2  0
2 yy '  3 x 2  2
3x 2  2
y' 
2y
at point of tangency
3 1  2
2
5
y'  
23 6
The normal line equation
1
y  y1  
dy
x  x1 
dx
1
y 3 
5
x   1

6
6
y  3   x  1
5
6 x  5 y  21  0

CALCULUS 1 v.1.0 by JOANNE MICHELLE B. AGUILAR, REE 106


UNIVERSITY OF NUEVA CACERES COLLEGE OF ENGINEERING AND ARCHITECTURE

 Learning Activity 14.1

Solution

1. Critical Numbers
f t   2t 3  3t 2  12t  7
f ' t   6t 2  6t  12
0  6t 2  6t  12

0  6 t2  t  2 
0  6t  1t  2
t  1, t  2
2.
𝑓’(𝑡) + - +

-2 1

f '  3  6 3  6 3  12  24 Positive (24) when t = -3 first interval


2

Negative (-12) when t = 0, middle


f ' 0  60  60  12  12
2
interval (24) when t = 2, last interval
Positive
f ' 2  62  62  12  24
2

3. According to the result of the first derivative test, (a) from positive to negative at zero, thus 𝑡 =
−2 is maximum, and (b) from negative to positive at 4, thus 𝑡 = 1 is minimum.
Solve for 𝑓(𝑡) using the function f t   2t 3  3t 2  12t  7

f  2   2 2   3 2   12 2   7  3


3 2
When x = -2
f 1  21  31  121  7  0 When x = 1
3 2

Therefore the maximum point occurs at P (-2, 3) and minimum point occurs at P (1, 0).

4. Substitute the critical numbers to the second derivative f ' ' t   12t  6

f ' '  2  12 2  6  18 Y’’ is negative thus the curve opens downward
f ' ' 1  121  6  18 Y’’ is positive thus the curve opens upward

5. Point of Inflection
f ' ' t   12t  6

CALCULUS 1 v.1.0 by JOANNE MICHELLE B. AGUILAR, REE 107


UNIVERSITY OF NUEVA CACERES COLLEGE OF ENGINEERING AND ARCHITECTURE

0  12t  6
1
t
2
Find 𝑓(𝑡) by substituting 𝑡 = − to the equation f t   2t 3  3t 2  12t  7
3 2
 1  1  1  1 27
f     2    3    12    7 
 2  2  2  2 2
Thus the point of inflection is at P (-1/2, 27/2).

 Learning Activity 15.1

1.

dV dh
Given:  100 Find:  100 when ℎ = 10
dt dt

Figure LA 15.1 Figure LA 15.2


V r 2 h Volume formula
3
By similar triangle method using figure LA 15.2 Replacing r in the volume formula using eqn. 1

eqn. 1 eqn. 2

CALCULUS 1 v.1.0 by JOANNE MICHELLE B. AGUILAR, REE 108


UNIVERSITY OF NUEVA CACERES COLLEGE OF ENGINEERING AND ARCHITECTURE

Differentiating both sides of eqn. 2 with respect to time

When = 100 and ℎ = 10


100  102 dh
16 dt
100 dh
100 
16 dt
dh
1600  100
dt
dh 16

dt 

2.

Figure LA 15.3 Figure LA 15.4


V r 2 h Volume formula
3
By similar triangle method

r 4 1
 
h 8 2
h
r eqn. 1
2

CALCULUS 1 v.1.0 by JOANNE MICHELLE B. AGUILAR, REE 109


UNIVERSITY OF NUEVA CACERES COLLEGE OF ENGINEERING AND ARCHITECTURE

Use eqn. 1 to express the volume in terms of h


V r 2h
3
2
 h
V   h
32

V h3 eqn. 2
12
Differentiating both sides of eqn. 2 with respect to time


V h3
12
dV  2 dh
 3h
dt 12 dt
dV h dh
2

dt 4 dt
dV
when h  6 and 2
dt
 62  dh
2
4 dt
dh 8 2
 
dt 36 9
3.

By ratio and proportion (similar triangle)

L 16
Figure LA 15.5 
Lx 6
Solving for L

6 L  16L  x 
16 x  16 L  6 L
8
L x
5

CALCULUS 1 v.1.0 by JOANNE MICHELLE B. AGUILAR, REE 110


UNIVERSITY OF NUEVA CACERES COLLEGE OF ENGINEERING AND ARCHITECTURE

Differentiating both sides with respect to time

8
L x
5
dL 8 dx

dt 5 dt
dx
at 5
dt
dL 8 dx

dt 5 dt
dx
at 5
dt
dL 8
 5
dt 5
dL
8
dt

4.

The volume of the cube with side x

V  x3
Differentiating both sides with respect to time

𝑉=𝑥
𝑑𝑉 𝑑𝑥
= 3𝑥
𝑑𝑡 𝑑𝑡
𝑑𝑥 1 𝑑𝑉
=
𝑑𝑡 3𝑥 𝑑𝑡
eqn. 1

Surface of the cube

A  6x 2
Differentiating both sides with respect to time

dA dx
 12 x eqn. 2
dt dt

dV
Substituting eqn. 1 to eqn. 2 with 2
dt

CALCULUS 1 v.1.0 by JOANNE MICHELLE B. AGUILAR, REE 111


UNIVERSITY OF NUEVA CACERES COLLEGE OF ENGINEERING AND ARCHITECTURE

dA 1 dV
 12 x 2
dt 3x dt
dA 1 8
 12 x 2 2 
dt 3x x
at x  8
dA 8
 1
dt 8

 Learning Activity 16.1

1. 9 = 𝑥 + 𝑦
𝑦 = 9 − 𝑥 eqn.1

𝑃 = 𝑥𝑦 eqn. 2

Substitute for y (eqn. 1 to 2)

𝑃 = 𝑥 𝑦 = 𝑥 ( 9 − 𝑥)

Differentiating both sides

𝑃 (𝑥) = 𝑥 (2)( 9 − 𝑥)(−1) + (1)( 9 − 𝑥)

0 = ( 9 − 𝑥)[ −2𝑥 + ( 9 − 𝑥)]

0= ( 9 − 𝑥)[ 9 − 3𝑥 ]

0= ( 9 − 𝑥)(3)[ 3 − 𝑥 ]

= 𝑥 = 9, 𝑥 = 3 two non-negative numbers

Thus the largest is P=108

2.

CALCULUS 1 v.1.0 by JOANNE MICHELLE B. AGUILAR, REE 112


UNIVERSITY OF NUEVA CACERES COLLEGE OF ENGINEERING AND ARCHITECTURE

500 = 5𝑥 + 2𝑦

2𝑦 = 500 − 5𝑥

𝑦 = 250 – 𝑥 eqn. 1

Area of the enclosure

𝐴 = 𝑥𝑦

Substitute for y (eqn. 1 to 2)

𝐴 = 𝑥𝑦

5
= 𝑥 250 − 𝑥
2

5
= 250𝑥 − 𝑥
2

Differentiating both sides

5
𝐴 (𝑥) = 250 − 2𝑥
2

0 = 250 − 5𝑥

0 = 5 (50 − 𝑥 )

𝑥 = 50; 𝑦 = 125

CALCULUS 1 v.1.0 by JOANNE MICHELLE B. AGUILAR, REE 113


UNIVERSITY OF NUEVA CACERES COLLEGE OF ENGINEERING AND ARCHITECTURE

3.

Total surface area of the box

48 = 𝑥 + 4 (𝑥𝑦)

4𝑥𝑦 = 48 – 𝑥

48 − 𝑥 48 𝑥 12 𝑥
𝑦= = − = − eqn. 1
4𝑥 4𝑥 4𝑥 𝑥 4

Total volume of the box

𝑉 = (𝑥)(𝑥)(𝑦) = 𝑥eqn.
𝑦 2

Substitute for y (eqn. 1 to 2)

𝑉 = (𝑥)(𝑥)(𝑦) = 𝑥 𝑦

12 1
𝑉=𝑥 − 𝑥
𝑥 4

1
𝑉 = 12𝑥 − 𝑥
4

Differentiating both sides

1
𝑉 (𝑥) = 12 − 3𝑥
4

3
0 = 12 − 𝑥
4

3
0= (16 − 𝑥 )
4

3
0= (4 − 𝑥)(4 + 𝑥)
4

𝑥 = 4; 𝑥 = −4

𝑥 = 4 𝑓𝑡 𝑎𝑛𝑑 𝑦 = 2 𝑓𝑡

CALCULUS 1 v.1.0 by JOANNE MICHELLE B. AGUILAR, REE 114


UNIVERSITY OF NUEVA CACERES COLLEGE OF ENGINEERING AND ARCHITECTURE

4.

Total surface area

3𝜋 = 𝜋𝑟 + (2𝜋𝑟)ℎ

2𝜋𝑟ℎ = 3𝜋 − 𝜋𝑟

3𝜋 − 𝜋𝑟
ℎ=
2𝜋𝑟

3 1
ℎ= − 𝑟 eqn. 1
2𝑟 2

Total volume of the cylinder

𝑉 = 𝜋(𝑟 )ℎ eqn. 2

Substitute for y (eqn. 1 to 2)

𝑉 = 𝜋(𝑟 )ℎ

3 1
𝑉 = 𝜋𝑟 − 𝑟
2𝑟 2

3 1
𝑉 = 𝜋𝑟 − 𝜋𝑟
2 2

Differentiating both sides

3 1
𝑉′(𝑟) = 𝜋 − 𝜋3𝑟
2 2

3
𝑉′(𝑟) = 𝜋(1 − 𝑟 )
2

3
𝑉′(𝑟) = 𝜋(1 − 𝑟)(1 + 𝑟)
2

3
0 = 𝜋(1 − 𝑟)(1 + 𝑟)
2

𝑟 = 1, 𝑟 = −1

𝑟 = 1, ℎ = 1

CALCULUS 1 v.1.0 by JOANNE MICHELLE B. AGUILAR, REE 115

You might also like